VBG vs ABG in the ED

Screen Shot 2018-09-27 at 11.40.11 AM.png

Written by: Emmanuel Ogele, MD (Cook County Stroger PGY-1) Edited by: Spenser Lang, MD (NUEM Alum '18) Expert commentary by: James Walter, MD


ABG’s vs VBG’s in the Emergency Department

  • Arterial blood gases (ABG’s) – blood sample taken directly from an artery used to gauge the metabolic environment, oxygenation, and ventilation status. Values such as pH, PCO2, PaO2, HCO3, and Base Excess obtained via ABG are considered the gold standard.

  • Venous Blood gases (VBG’s) – blood sample taken from either peripheral or central veins –can serve as an alternative to an ABG when evaluating patients with metabolic and respiratory disturbances.

    • Historically, values obtained via VBG have been criticized for a perceived lack of accuracy in all domains.

    • However, VBGs carry less risk of vascular injury, nerve damage, and cause much less pain to the patient along with lower risk for accidental needle-sticks as compared to ABGs


  • So the question remains – are values (such as pH, PCO2, and HCO3) truly disparate enough between ABG’s and VBG’s to actually change clinical practice?

    • Increasing data shows that for most clinical indications, data from VBG correlates well, and are just as useful as that from ABG.[1-4]

      • Zeserson et. al. conducted a prospective cohort study of 156 critically ill patients in the ED and ICU setting to evaluate the correlation between pH and pCO2 when derived from ABG vs VBG with added pulse oximetry for estimating PaO2 and concluded that arterial and venous pH and PCO2 had good correlation.

      • Byrne et al conducted a meta – analysis of 1768 subjects from 18 individual studies and found that peripheral VBG correlates well with ABG with respect to pH but found an unacceptably wide 95% prediction interval when looking at the pCO2.

      • A review article by Kelly AM summarized data comparing ABG and peripheral VBG variables in ED all-comers also concluded that venous pH had sufficient agreement however concluded with a word of caution: there is no data to support that this correlation is maintained in shock states.

    • Several studies have looked at the correlation between values obtained with VBG and compared them to ABG. These are summarized in Table 1.

Screen Shot 2018-10-10 at 6.26.27 PM.png

Table 1. Correlation of VBG to ABG values
** Widest limit of agreement from any single study included in the meta-analysis


  • For most parameters, there is good correlation. However, there are a few important scenarios that may be exceptions. Not surprisingly, the major exception is PO2; venous PO2 readings do not correlate well with arterial PO2. A workaround to this limitation is to estimate arterial oxygenation using SpO2.

  • The VBG analysis plus SpO2 provided accurate information on acid–base, ventilation, and oxygenation status for patients in undifferentiated patients ED and ICU.[2]

  • VBGs are acceptable to use in working up common conditions like COPD and DKA.[5,6] New data could potentially broaden the list of indications for VBG instead of ABG  

    • Ma OJ et al. conducted a prospective trial looking at the utility of ABG in patients presenting to the ED with suspected DKA and found that ABG analysis changed management of DKA 1% of the time and concluded that VBGs are a viable substitute.

Conditions that may affect the reliability of VBG

  • Hypercapnia.

    • When comparing VBG and ABGs, the average difference in CO2 reading was 5.7 mmHg. [1]

    • However, the limits of agreement (-17.4 to +23.9) in this study are too wide to allow reliable quantification of PCO2.

    • In sum, if you need a precise PCO2 number for clinical decision making, a traditional ABG is preferable.

    • One such scenario where a true PCO2 can be useful is evaluating for acute hypercapneic respiratory failure; however, a VBG still has some utility.

      • In the prospective study by AM Kelly 7 a PCO2 value above 45mmHg had a 100% sensitivity for true hypercapnia. This makes a VBG PCO2 value useful in screening for hypercapnia. 5

  • Shock Pearls

    • VBGs show increased discordance from ABGs in hypotensive patients.[8]

    • pH and PCO2 values may be wildly disparate in patients with severe circulatory failure.[9]

    • In sum, venous blood gases may be increasingly inconsistent with arterial blood gases in patients with increasing degrees of shock. No definitive data exists yet to tell us if VBGs are sufficient to replace ABGs in shock states.

  • Mixed Acid Base Disorders

    • There is insufficient evidence to confirm reliability of VBGs in these cases

 

In summary, VBGs can be used as a reliable alternative to ABGs in many clinical cases. The patients’ benefits of a VBG vs ABG are obvious – decreased pain, complications, and time. Clinical judgment must be used in deciding when to the substitute a VBG for a more traditional ABG. The evidence is mixed, and even non-existent in some clinical scenarios. In the future, noninvasive methods of evaluation, such as transcutaneous PCO2 monitoring and ETCO2, could allow for accurate for non-invasive and monitoring of the metabolic milieu.


Expert Commentary

ABGs vs VBGs in the Emergency Department: Expert Commentary 

Thank you for the opportunity to share some thoughts on this topic. The ABG vs VBG debate has been the source of a lot of discussion and at times disagreement between EM and IM. I am hopeful that we are starting to reach consensus on their respective advantages, disadvantages, and indications. When deciding on which test to obtain, here are a few questions to ask yourself:


1. What is my clinical question?

Diagnostic tests should be performed to answer a specific clinical question. Defining this question will help ensure you order the correct test, or perhaps appropriately order no test at all. For blood gas sampling this question might be: “Does my patient with a COPD exacerbation have significant hypercapnia?”; “Is my patient appropriately compensating for his metabolic acidosis?”; “Is my hyperglycemic patient acidotic?” If you can’t articulate a specific question, or if the answer to that question is unlikely to change your management (i.e., a question of “is my patient acidotic?” for a 70-year-old with urosepsis whose blood pressure has responded to 1L of fluid and looks well), then you can probably save your patient an unnecessary blood draw and avoid blood gas sampling altogether. This is certainly an issue for us in the ICU. Patients with arterial lines will have standing Q6hr ABG orders for 2 days before anyone asks if those blood draws are actually changing our management. Don’t order an ABG or VBG just because a patient has sepsis, or they have COPD, or you are “screening for badness.” Using a POC or rapid VBG with a metabolic profile to rapidly obtain lab values for patients presenting to the ER is reasonable. Outside of this situation, try to make sure you are asking a specific question and that answering that question is likely to change what you do.


2. Am I screening for hypercapnia?

If your clinical question is, “is my patient hypercapnic?” then a VBG is a great test. As noted above, a PvCO2 < 40 mmHg excludes hypercapnia. This can be an extremely helpful in the rapid workup of altered mental status and many other common presenting conditions. 


3. How accurate do I need my PCO2 value to be?

If the answer to this question is “not that accurate” then a VBG is probably fine. Having a rough estimate of PCO2 levels is usually adequate for the management of mild-moderate DKA, COPD exacerbations, and many other conditions managed in the ED. While a PvCO2 value of 18 mmHg or 75 mmHg may not exactly correlate with what you find on a PaCO2, they are abnormal enough to give you a good general sense of things.

If you are interested in performing a more refined blood gas analysis such as determining the chronicity of a respiratory acidosis, measuring shunt fraction, or accurately quantifying a hypercapnic patient’s true PCO2 then you probably need an ABG. As noted above, the correlation between PaCO2 and PvCO2 is often poor. 

 

4. Am I assessing oxygenation?

At times, obtaining a reliable SpO2 can challenging especially in patients with PAD, scleroderma, or shock. If you need an accurate assessment of oxygenation then you need an ABG. PvO2 values do not correlate well at all with PaO2.

 

5. Is my patient in shock?

As noted above, VBGs are much less accurate in shock. Unfortunately, this is where we are often most interested in frequent blood gas analysis. In the ED, I think ABGs are most useful (and underused) in critically ill acidotic patients who may or may not have appropriate respiratory compensation. This determination is hard to make on clinical grounds alone (i.e. the signs of early respiratory muscle fatigue can be subtle) and identifying fatigue may well change your management (pushing you to earlier NIV or mechanical ventilation). I would hesitate to solely rely on VBGs in this setting especially for patients in overt shock.

 

A few other points:

  • I do think the risks of an ABG as stated above and in other reviews (for example, https://lifeinthefastlane.com/ccc/vbg-versus-abg/) are overstated. A competent clinician should be able to obtain an ABG from a radial artery in a matter of seconds. If there are any concerns regarding anatomy or first stick accuracy, the use of a vascular ultrasound probe can remove any guess work from finding the best arterial access site. ABGs do require an extra needle stick for patients so clinicians should be discerning about their use. However, if one is indicated they shouldn’t be avoided for fear of causing a pseudoaneurysm or major bleeding. Compared to innumerable other invasive procedures and diagnostic tests performed in the ED, ABGs are pretty benign. For some reason, they are still frequently described like thoracotomies.

  • Remember the following rough corrections

    • Venous pH is 0.03 lower than arterial pH (venous pH 7.27 = arterial pH 7.3)

    • Venous PCO2 is 6 mmHg higher than arterial PCO2, but with wide variability; in general, difficult to predict arterial PCO2 from venous PCO2 (although a PCO2 can still be useful, as noted above).

James_Walter-44.png
 

James Walter, MD

Pulmonary and Critical Care, Northwestern Medicine

Medical Director of the Northwestern Lung Rescue Program


How to Cite This Post

[Peer-Reviewed, Web Publication]   Ogele E, Lang S (2018, October 15). VBG vs ABG in the ED.  [NUEM Blog. Expert Commentary by Walter J]. Retrieved from http://www.nuemblog.com/blog/vbg-abg


Other Posts You May Enjoy


 Resources 

  1. Kelly AM. Review article: Can venous blood gas analysis replace arterial in emergency medical care? Emerg Med Australas. 2010 Dec;22(6):493-8. doi: 10.1111/j.1742 6723.2010.01344.x. Review. PubMed PMID: 21143397

  2. Zeserson E, Goodgame B, Hess JD, Schultz K, Hoon C, Lamb K, Maheshwari V,Johnson S, Papas M, Reed J, Breyer M. Correlation of Venous Blood Gas and Pulse Oximetry With Arterial Blood Gas in the Undifferentiated Critically Ill Patient. J Intensive Care Med. 2016 Jun 9.

  3. Byrne, A. L., Bennett, M., Chatterji, R., Symons, R., Pace, N. L. and Thomas, P. S. (2014), Peripheral venous and arterial blood gas analysis in adults: are they comparable? A systematic review and meta-analysis. Respirology, 19: 168–175. doi:10.1111/resp.12225

  4. Kelly AM, McAlpine R, Kyle E. Venous pH can safely replace arterial pH in the initial evaluation of patients in the emergency department. Emerg Med J. 2001 

  5. McCanny, Venous vs arterial blood gases in the assessment of patients presenting with an exacerbation of chronic obstructive pulmonary disease.

  6. Ma OJ, Rush MD, Godfrey MM, Gaddis G. Arterial blood gas results rarely influence emergency physician management of patients with suspected diabetic ketoacidosis. Acad Emerg Med. 2003 Aug;10(8):836-41. PMID 12896883

  7. Anne-Maree Kelly, Elizabeth Kyle, Ross McAlpine, Venous pCO2 and pH can be used to screen for significant hypercarbia in emergency patients with acute respiratory disease, In The Journal of Emergency Medicine, Volume 22, Issue 1, 2002, Pages 15-19, ISSN 0736-4679, https://doi.org/10.1016/S0736-4679(01)00431-0.

  8. Shirani F, Salehi R, Naini AE, Azizkhani R, Gholamrezaei A. The effects of hypotension on differences between the results of simultaneous venous and arterial blood gas analysis. Journal of Research in Medical Sciences : The Official Journal of Isfahan University of Medical Sciences. 2011;16(2):188-194

  9. Adrogué HJ, Rashad MN, Gorin AB, Yacoub J, Madias NE. Assessing acid-base status in circulatory failure. Differences between arterial and central venous blood. N Engl J Med. 1989 May 18;320(20):1312-6.

Posted on October 15, 2018 and filed under Pulmonary.

Supratherapeutic INR

Screen Shot 2018-09-27 at 11.32.54 AM.png

Written by: Luke Neill, MD (NUEM PGY-3) Edited by: Logan Weygandt, MD (NUEM Alum '17) Expert commentary by: Abbie Lyden, PharmD BCPS


Expert Commentary

This is an excellent post on the management of supratherapeutic INR in patients taking vitamin K antagonist therapy – and as you described, there is not a one-size-fits-all approach. Warfarin is notorious for being one of the most difficult medications to manage based on narrow therapeutic index, variable dose response, clinically significant diet- and drug- drug interactions, delayed onset and offset of action and the need for frequent monitoring. Fortunately, warfarin does have an antidote in vitamin K. Yet, the choice of when to administer this antidote (and weaknesses of said antidote), along with other therapies including prothrombin complex concentrates (PCCs) and fresh frozen plasma, are not straightforward and depend upon a number of factors. In addition to your excellent teaching points, I have outlined some additional considerations below.

Life-threatening bleeding

In the setting of life-threatening bleed, guidelines dictate our therapeutic approach, which involves holding warfarin and administering 4-factor PCC and intravenous vitamin K (10mg slow infusion over 20-60 minutes)1.

Minor bleeding

The management of life-threatening bleeding is clear and requires aggressive therapy. But how do we manage the patient who has a moderately elevated INR but only a minor bleed? In the setting of minor bleeding (such as intermittent epistaxis), the goal is restore the INR to target range, without leading to subtherapeutic anticoagulation, thus introducing the risk of thrombosis. There is general consensus regarding holding a dose of warfarin in these scenarios but the choice to administer vitamin K has been debated. The choice of approach should depend upon the perceived risk of bleeding, extent of bleed, site of bleed, INR level (and trend in INR), comorbidities (including indication for anticoagulation) and risk of thromboembolism. The downsides to administering vitamin K are worth mentioning as they are sometimes superseded by our focus on providing active treatment (unquestionably necessary in the case of life-threatening bleed).  Excessive vitamin K dosing may result in warfarin resistance for 1-2 weeks which may require extensive bridging therapy once anticoagulation is restarted. For patients with high thromboembolic risk, poor adherence to medications, higher INR goals or comorbidities, this can become complicated and is certainly not without risk.

Elevated INR without bleeding

The 2012 ACCP guidelines recommend administration of oral vitamin K (2.5-5mg) to patients without an active bleed who have an INR>10 (1). Other experts and the 2008 ACCP guidelines use a more conservative cutoff of 9 (2). For those patients with INRs between 4.5 and 10 without evidence of bleeding, the 2012 ACCP guidelines suggest against the routine use of vitamin K. In this post, you make a great point regarding the management of INRs 5-9 without bleeding. That is, the administration of vitamin K may or may not be administered, depending on the risk of bleeding. Low dose vitamin K administration should be more strongly considered in patients with high risk bleed (elderly, prior bleed) and lower risk of thromboembolism. A retrospective review of 633 patients with elevated INR >6 identified risk factors for slow spontaneous lowering of supratherapeutic INR, including older age, higher index INR, lower warfarin maintenance dose, decompensated heart failure and active cancer (3). Knowledge of these risk factors may help guide decision-making when considering whether or not to administer vitamin K.

Urgent Surgery/Procedure

Patients meriting further discussion are those taking vitamin K antagonists who require urgent (same day) surgeries or invasive procedures. These patients are managed in a similar fashion to those with life threatening bleeding – that is to say, vitamin K (10mg IV) and 4-factor PCC. Of note, for those patients who can wait 24 hours, low dose vitamin K (1-2.5mg PO) is generally adequate for INR reversal. In these cases, 4-factor PCC and intravenous vitamin K can be avoided.

Specific Treatments

With regards to the specific treatments to reverse supratherapeutic INR, some points to keep in mind:

  1. Vitamin K (phytonadione): typically administered intravenous or orally.

  • For life-threatening bleed, intravenous vitamin K is preferred due to the faster onset of action (which is still delayed, ~3-8 hours) but faster than oral (onset ~24 hours)

  • Subcutaneous administration is generally avoided if possible due to erratic absorption

  • Intramuscular administration is generally avoided due to the risk of hematoma in an anticoagulated or overanticoagulated patient

2. Prothrombin complex concentrate (PCC):

  • 4-factor PCCs (factors II, VII, IX, X): preferred first line therapy for life-threatening bleed

  • Activated PCC (aPCC): factor VII is mostly present in the activated form, which is potentially more prothrombotic than unactivated PCC

  • Some PCC products contain heparin and should NOT be administered in a patient with a history of heparin-induced thrombocytopenia

In short, patients with life-threatening bleed taking vitamin K antagonist therapy require urgent evaluation and treatment with PCC and IV vitamin K. Treatment with PCC is paramount as INR can be corrected within 30 minutes, as opposed to several hours after IV vitamin K administration (onset dependent upon liver synthesis of new coagulation factors). For those patients with supratherapeutic INR without bleeding and patients with minimal bleed, a more gentle approach is indicated, which involves omission of warfarin dose +/- low doses of oral vitamin K to ensure correction of INR and prevention of bleeding but not subtherapeutic anticoagulation. Lastly, it is important to identify if there are additional explanations for the supratherapeutic INR by verifying if the patient was taking the appropriate dose of warfarin or whether they have recent dietary changes or new medications which may interact. If therapy is to be resumed, these questions are particularly helpful when deciding how and when to restart anticoagulation. 

  1. Holbrook A, Schulman S, Witt DM, et al. Evidence-based management of anticoagulant therapy: Antithrombotic Therapy and Prevention of Thrombosis, 9th ed: American College of Chest Physicians Evidence-Based Clinical Practice Guidelines. Chest 2012;141(2 Suppl):e152S.

  2. Ansell J, Hirsch J, Hylek E, et al. Pharmacology and management of the vitamin K antagonists: American College of Chest Physicians Evidence-Based Clinical Practice Guidelines (8th Edition). Chest 2008;133(6 Suppl):160S.

  3. Hylek EM, Regan S, Go AS, Hughes RA, Singer DE, Skates SJ. Clinical predictors of prolonged delay in return of the international normalized ratio to within the therapeutic range after excessive anticoagulation with warfarin. Ann Intern Med. 2001;135(6):393.

Abbie.png
 

Abbie Lyden, PharmD BCPS

Clinical Pharmacist, Emergency Medicine | Associate Professor, Pharmacy Practice

Residency Program Director, PGY2 EM Pharmacy

Northwestern Memorial Hospital


How To Cite This Post

[Peer-Reviewed, Web Publication]   Neill L, Weygandt L (2018, October 8). Supratherapeutic INR.  [NUEM Blog. Expert Commentary by Lyden A]. Retrieved from http://www.nuemblog.com/blog/supratherapeutic-inr


Other Posts You May Enjoy

Posted on October 8, 2018 and filed under Hematology.

End Tidal CO2 in Cardiac Arrest

Screen Shot 2018-09-27 at 11.16.07 AM.png

Written by: Alex Herndon, MD (NUEM PGY-2) Edited by: Andrew Moore, MD (NUEM Alum '18) Expert commentary by: Seth Trueger, MD, MPH

Introduction:

ER, Grey’s Anatomy, House, Chicago Med, The Good Doctor - across the nation millions tune in to their favorite medical dramas hoping to get a glimpse at what it’s like to be in the business of saving lives. As a newly minted emergency medicine intern, my eye caught on to the most recent addition, The Resident, intrigued by how the show would portray the medical profession. While watching and cringing at dramatized incidences of medical nonsense, one scene particularly stood out.

It was day one of residency and the new intern was leading the resuscitation of a patient who suddenly arrested. Fast-forward, the intern achieves return of spontaneous circulation, however he is immediately chastised by his senior resident who states, “her end-tidal CO2 was less than 15 for the entire code…”

As the credits rolled I was left agreeing with critic Dr. Esther Choo in that “this show feels like a most unfortunate and untimely addition to the medical drama genre”, given it is the least accurate medical show I’ve watched to date, except for this fleeting reference to End Tidal CO2 (ETCO2).

 

A Review of ETCO2 and its Applications:

Traditionally, ETCO2 has been used in order to assess proper endotracheal tube placement. Approximately 25 years ago anesthesiologists began using ETCO2 capnography because it was revealed that approximately 93% of anesthesia errors could have been prevented with additional capnography monitoring. In particular, the sensitivity of color change of colorimetric devices can be faulty at low concentrations of CO2, a particular concern when there is presumed decreased CO2 released from the lungs due to decreased cardiac output during an arrest, or if the ET tube is placed within the esophagus. The addition of capnography not only reinforces that the ET tube is properly placed, but its use has been extrapolated to indirectly assess cardiac output. (1)

 

The additional data ETCO2 supplies can be used in two key ways:

 

1. To assess quality of chest compressions

In cardiac arrest, ETCO2 waveform, while performing CPR, can serve as an indirect measurement of blood flow generated by chest compressions. The height of the ETCO2 waveform during CPR has been used as an indirect measure of adequate chest compressions, helping those involved in resuscitation monitor the effectiveness of their compressions in real time. In the awake adult, normal cardiac index lies between 2.5-4 L/min/m2, with an ETCO2 of 35-45 mmHg. On average during CPR, if adequate chest compressions are being delivered a cardiac index of 1.6-1.9 L/min/m2 can be generated, which correlates with ETCO2 pressures of 20mmHg.(1) ACLS guidelines define high quality chest compressions as achieving ETCO2 pressures of at least 10-20 mmHg. As rotating medical professionals deliver chest compressions, ETCO2 can be used to determine if they need to be deeper, if there is performer fatigue, or if there are other factors that might be inhibiting the ability to maintain ideal cardiac output outside of ineffective chest compressions. All in all, it provides a more accurate assessment of chest compression adequacy than visual estimation of compression depth.

 

2. To help predict return of spontaneous circulation (ROSC)

Numerous studies have shown that abrupt increases in ETCO2 pressures exceeding 10 mmHg that remain higher than preceding values suggest an increase in cardiac output and is indicative of ROSC, hence the incorporation of such measures in ACLS guidelines.(1) Patients with values less than 10 mmHg are more likely to die during CPR, and those with values greater than 10 during CPR were more likely to get ROSC. (1) However when comparing a mere 10 mmHg ETCO2 pressure to the minimal normal ETCO2 pressure of 35 mmHg, it can be difficult to argue 10 mmHg is enough. Multiple studies have aimed to drive this number up, in particular showing ETCO2 pressures higher than 16mHg were significantly associated with survival from CPR in the emergency department. However, the use of an absolute ETCO2 value was limited by the cause of cardiac arrest. Average ETCO2 pressures that achieved ROSC widely varied depending on whether cardiac arrest was purely cardiac versus pulmonary in etiology. (1) A 2015 meta-analysis of ETCO2 values associated with ROSC showed, on average, patients with ROSC after CPR had an average ETCO2 level of 25 mmHg, significantly higher than the current recommended 10 mmHg threshold.(2) Other than the aforementioned minimal ETCO2 threshold, it is important to follow ETCO2 trends, looking for the sudden increase in ETCO2 and maintenance of elevated levels associated with ROSC.

 

Furthermore, other studies have attempted to show how ETCO2 can be a tool in deciding to terminate CPR when ROSC isn’t achieved. One study from 1997 reported that ETCO2 less than 10 mmHg at the 20 minute mark is predictive of non-survivability in outside-hospital cardiac arrest patients thus should lead to terminating resuscitation efforts.(3) Current studies of use of ETCO2 in outside hospital arrest trends have shown 3-5 minutes of ETCO2 <10 mmHg are associated with a bad prognosis and has been used to terminate in-field resuscitation efforts.(4)

 

Next Steps:

While more data is needed in order to potentially reset the ETCO2 threshold used to assess adequate CPR and ultimately long-term survival post-arrest, others are looking at alternate applications of ETCO2. In 2016 Wang et al studied whether or not ETCO2 values could be used as a predictor of survival when looking at in-hospital versus outside-hospital cardiac arrests. They found that an initial ETCO2 level was predictive of not only sustained ROSC, but also survival to discharge.(2) Others have looked at the use of ETCO2 to determine potential effectiveness of defibrillation. It was found that ETCO2 less than 7 mmHg never resulted in effective shocks, whereas shocks delivered with ETCO2 greater than 45 mmHg were always successful. Ultimately the authors attributed the success to performing high quality chest compressions prior to defibrillation.(5)

 

Limitations of ETCO2 Capnography and Conclusions:

Similar to how ETCO2 can vary given the cause of cardiac arrest, ETCO2 can be influenced by other factors, thus altering how physicians interpret capnography. In particular, no studies have assessed the effect or epinephrine or sodium bicarbonate on ETCO2. Further complicating matters, the majority of cardiac arrest patients end up intubated, thus ventilator settings or over bagging can influence expired CO2 levels creating yet another confounding factor. These subtleties regarding ETCO2 need further exploration.   

Next time you are waiting for that outside-hospital cardiac arrest to roll through the Emergency Department entrance, arm yourself with ETCO2 capnography not only to aid your resuscitative efforts, but also help with your decision-making along the way. Its usefulness extends beyond simply achieving ROSC and has the potential to prognosticate whether patients will not only survive but thrive.


Expert Commentary:

Thanks for this nice overview of the data behind quantitative waveform ETCO2 in arrest. While it’s not the only tool in our armamentarium, it certainly can be helpful in assessing whether compressions are being effectively done, which can help the compressor modify their technique or location, add an element of motivation for the compressor, and help identify when the compressor is tiring out so we can switch someone else in before the official 2 minutes is up. Similarly, if there’s a big jump upward in the ETCO2 (eg, from 12 to 35), it’s reasonable to deviate from the usual 2 minutes and jump to a pulse check.

Using low ETCO2s is helpful in identifying futile codes; the general rule is that if the ETCO2 is consistently <10 after 20 minutes of well-done ACLS, the patient is very unlikely to come back. I find it important to point out that this is not a sensitive test, however, and many arrested patients will continue to have ETCO2 over 10 during long codes. I try to not get too focused on the ETCO2 as the only marker of when to terminate resuscitative efforts. Rather, it can help make a hard decision easier in some cases, but like most things, it’s not a magic bullet.

One other caution: don’t be mislead by a flat ETCO2 waveform during an arrest. If you don’t see any waves, then the airway is not in and either replace it or confirm intubation by other means (eg VL or gentle bougie insertion to holdup).

Here is a short screencast I put together back in 2013 on the 3 major uses of ETCOS: ETT confirmation, arrest, and monitoring in procedural sedation:

https://player.vimeo.com/video/57510987

seth trueger.png
 

Seth Trueger, MD, MPH

Assistant Professor of Emergency Medicine, Northwestern University


How To Cite This Post

[Peer-Reviewed, Web Publication]   Herndon A, Moore A (2018, October 1). End Tidal CO2 in Cardiac Arrest.  [NUEM Blog. Expert Commentary by Trueger NS]. Retrieved from http://www.nuemblog.com/blog/ETCO2


Other Posts You May Enjoy


Resources

  1. Kodali, B. Urman, R. Capnography during cardiopulmonary resuscitation: Current evidence and future directions. J Emerg Trauma Shock. 2014 Oct-Dec: 7(4): 332-340. Doi: 10.4103/0974-2700.142778

  2. Wang, AY. Initial end-tidal CO2 partial pressure predicts outcomes of in-hospital cardiac arrest. Am J Emerg Med. 2016 Dec;34(12):2367-2371. doi: 10.1016/j.ajem.2016.08.052.

  3. Morshedi, B. The role of ETCO2 in termination of resuscitation. J Emerg Med Services. 2017 Dec. <http://www.jems.com/articles/print/volume-42/issue-12/features/the-role-of-etco2-in-termination-of-resuscitation.html>

  4. Venkatesh, H. Keating, E. Can the value of end tidal CO2 prognosticate ROSC in patients coding into emergency department with an out-of-hospital cardiac arrest. Emerg Med J. 2017 Mar; 34(3): 187-189. doi: 10.1136/emermed-2017-206590.1

  5. Savastano, S et al. End-tidal carbon dioxide and defibrillation success in out-of-hospital cardiac arrest. Resuscitation. 2017 Dec;121:71-75. doi: 10.1016/j.resuscitation.2017.09.010.

 

Posted on October 1, 2018 and filed under Pulmonary.

Phytophotodermatitis

Written by: Luke Neill, MD (NUEM PGY-3) Edited by: Mitali Parmar, MD (NUEM Alum '18) Expert commentary by: Charles Pearce, MD

Written by: Luke Neill, MD (NUEM PGY-3) Edited by: Mitali Parmar, MD (NUEM Alum '18) Expert commentary by: Charles Pearce, MD


Case

A 7 yo female with no significant past medical history presents with a two-day history of worsening asymmetric rash on the left neck, upper middle chest, right thigh, and the dorsal aspect of both hands. The rash is described as a painful, burning, 3/10 pain and non-pruritic. She denies any history of allergies, especially any allergies to outdoor plants or foods, and denies using any new types of lotions, sunscreen, or other new chemical products. She denies history of trauma. She denies fevers, chills, or recent infections. Denies arthralgias, myalgias, fatigue, or weakness. Denies SOB or chest pain. Denies peeling of her skin or blistering.  She takes no medications.

She is currently attending an outdoor day-time summer camp and her mother first noticed the rash after picking her up at the end of camp two days ago. The mother, due to concern for possible physical abuse would like to know your opinion on its cause. You ask to speak to patient in private and she adamantly denies any physical abuse. You check her vitals.

Vitals/Exam: 

HR: 65   BP:120/70 Temperature: 96.4 RR:16 O2:100

General: patient well appearing, watching TV

Skin: Multiple small areas of blotchy erythema over left neck, upper middle chest, right thigh, and dorsal aspect of hands in different patterns with no symmetry. Some appear to represent a hand print. The rash on the chest also appears to streak vertically.

Head: normocephalic, atraumatic

HEENT: oral mucosa moist, PERRL, EOM intact, TMs clear bilaterally

Neck: supple, trachea midline

Cardiovascular: regular rate/ rhythm, no murmurs, rubs, or gallops

Chest: non-tender

Pulmonary: clear to auscultation bilaterally

GI: non-distended, soft, non-tender 

MSK: no deformities

Neuro: CNII-XII intact, strength/sensation grossly intact

Skin Exam: 

Screen Shot 2018-09-14 at 10.48.31 PM.png

Labs: CBC and BMP Unremarkable

Clinical Course:

In consultation with dermatology, patient revealed that she recently made limeade at her summer camp about 2 days prior to arrival, and had been out in the sun for outdoor activities. Given this history and appearance of the rash, the patient was diagnosed with phytophotodermatitis, and discharged home with instructions to stay indoors. She was seen in clinic after 5 days, and had developed multiple blisters over her hands, chest and thighs. The rash subsequently completely resolved in 2 weeks time.


Phytophotodermatitis

Pathophysiology:

Phytophotodermatitis, also known as “Lime Disease” or “Margarita Photodermatitis” is a phototoxic inflammatory eruption of the skin that occurs due to contact with light-sensitizing botanical substances and subsequent exposure to UV-A radiation. Interestingly, the reaction of phytophotodermatitis is actually independent of the immune system.

But How?

When furocoumarins, the photosensitizing chemical compound produced by certain plants, are struck by a photon in the UV-A range of (320-400), energy is absorbed causing the formation of an excited state from ground state. When the furocoumarin returns to ground state, energy is released in the form of heat and fluorescence, leading to both DNA and RNA damage and cell death.

Furocoumarin is present in:

  • Limes

  • Figs

  • Parsley

  • Celery

  • Carrots

Timing:

  • Skin eruption typically begins 24 hours after sun exposure

Manifestation:

  • Burning erythema with blistering

  • Post-inflammatory hyperpigmentation lasting weeks to months. 

Progression:

  • Eruption peaks at approximately 48-72 hours

 

Below is an image illustrating the progression in days, of a person with phytophotodermatitis of the hand:

Epidemiology:

As phytophotodermatitis occurs independent of the immune system; any race, sex, or age group may be affected. However, it does appear that produce workers in grocery stores are at a much higher risk than the general population. A 1986 study showed that in one un-named nationwide grocery store chain, a randomly selected sample of all its stores revealed phytophotodermatitis occurrence in 13 of 17 states, with occurrence in 26% of the produce workers surveyed. In this instance, it was thought to be due to celery stock with higher levels of endogenous furocoumarin.

Treatment/Prognosis:

The prognosis of phytophotodermatitis is very good with proper identification and elimination of the offending plant. Patients who are affected should stay indoors avoiding UV-A rays to allow the dermatitis to self-resolve. 

Note: This is not the first time phytophotodermatitis has mimicked child abuse. A 1985 study looked at two separate cases of children who were initially thought to have hyper-pigmented skin lesions suggestive of child abuse and were later given a final diagnosis of phytophotodermatitis.

Takeaways/Learning Points:

  • Consider phytophotodermatitis in your differential for rash in summer months.

  • Although we ask about allergies, consider asking about recent food exposure.

  • Don’t spill your drink!


Expert Commentary

Great overview of phytophotodermatitis!  This is certainly a fascinating and easily missed phenomenon given its relative rarity, variable pattern of presentation and, on occasion, insidious exposures. In the Midwest, we are seeing additional exposure risks to phytophotodermatitis in the form of Wild Parsnip (known colloquially as poison parsnip), an invasive plant that was introduced from Europe over a century ago and whose range has continued to expand.  This expansion has lead to Department of Natural Resource and local media in a number of states working to educate the public as well as local and regional medical facilities about the potential threat.

As with lime juice, exposure to the sap of Wild Parsnip can be potentially non-apparent to the patient and the clinician.  In fact, the pattern of burns from Wild Parsnip are as variable as the methods of exposure- from linear lesions from unwittingly brushing against a plant to extensive hand/forearm involvement from attempted manual removal of a plant to a speckled burn pattern from mechanical disruption of the plant (think weed whacker or lawn mower).  This variability obviously heightens the diagnostic difficulty and uncertainty. 

Another consideration and as noted in your excellent review, phytophotodermatitis is a burn resultant of chemically induced cell death by cross linkage of the furan ring with pyrimidine bases in the presence of UV light.  And recognition of this phenomenon as a chemical burn does have bearing on management. A discussion with or referral to burn centers may be warranted, as with any burn, if there is significant TBSA involvement or depending on body area impacted.  And looking for and warning patients of the potential for super-infection is imperative.

As a ER physicians in rural Wisconsin, our practice sees a couple handfuls of phytophotodermatitis cases each summer.  Our local communities (namely farmers) are well aware of Wild Parsnip and the simple prevention of avoidance and washing exposed areas to remove sap.  As more and more people (hopefully) continue to venture out of cities and explore the hiking, biking and nature trails of the rural Midwest, broadening public awareness of a potentially painful exposure matters and so thank you for the chance to respond to your fantastic blog!

Picture1.png
 

Charles Pearce, MD NUEM ’14

Madison Emergency Physicians


How to Cite This Post

[Peer-Reviewed, Web Publication]   Neill L, Parmar M (2018, September 24). Phytophotodermatitis.  [NUEM Blog. Expert Commentary by Pearce C]. Retrieved from http://www.nuemblog.com/blog/phytophotodermatitis


Other Posts You May Enjoy

References

  1. Berkley SF, Hightower AW, Beier RC, et al. Dermatitis in grocery workers associated with high natural concentrations of furanocoumarins in celery. Ann Intern Med. 1986 Sep. 105(3):351-5. 

  2. Coffman K, Boyce WT, Hansen RC. Phytophotodermatitis simulating child abuse. Am J Dis Child. 1985 Mar. 139(3):239-40.

  3. Marcos LA, Kahler R. Phytophotodermatitis. Int J Infect Dis. 2015 Sep. 38:7-8. 

  4. Smith E, Kiss F, Porter RM, Anstey AV. A review of UVA-mediated photosensitivity disorders. Photochem Photobiol Sci. 2011 Dec 16. 11(1):199-206. 

  5. Becker, M. (2017). Phytophotodermatitis Rash [Digital image]. Retrieved November 10, 2017, from https://findadermatologist.com/healthinfo/phytophotodermatitis-limes-sunshine-don’t-mix.

  6. Kid, K (June 3, 2015) Phytophotodermatitis From Exposure to Lime Juice [Digital image]. Retrieved November 10, 2017, from https://upload.wikimedia.org/wikipedia/commons/thumb/b/be/Phytophotodermatitis_from_esposure_to_lime_juice.jpg

 

 

Posted on September 24, 2018 and filed under Dermatology.

qSOFA and SIRS

Screen Shot 2018-08-09 at 12.20.36 AM.png

Written by:  Alex Ireland, MD (NUEM PGY-3) Edited by:  Kim Iwaki, MD (NUEM Alum '18) Expert commentary by: Benjamin Schnapp, MD


Sepsis, a maladaptive host-response to infection, is a leading cause of morbidity and mortality within the healthcare system. We have known about and discussed this disease for decades, but recently have begun to alter our criteria for its diagnosis. Since 1991, we have categorized sepsis as a derangement in physiologic or laboratory parameters caused by a host’s systemic inflammatory response to an infection [1]. Two of 4 criteria, listed below, must be met in addition to a suspected source of infection. Sepsis complicated by end-organ dysfunction was deemed severe-sepsis. And sepsis-induced hypotension refractory to adequate fluid resuscitation was deemed septic shock.

Source: http://www.admitologist.com/wp-content/uploads/2015/12/Sepsis-Info.jpg

Source: http://www.admitologist.com/wp-content/uploads/2015/12/Sepsis-Info.jpg

Using these definitions, multiple studies have resulted in consensus guidelines that reduce morbidity and mortality, including early adequate fluid resuscitation, obtaining blood cultures before antibiotic therapy, administration of broad-spectrum antibiotics within 1 hour of diagnosis, and use of norepinephrine as a first-line vasopressor to maintain MAP > 65 mmHg2.

Recently, due to presumed limitations in the original definition, a task force was convened and a new definition was proposed. They define sepsis as life-threatening organ dysfunction caused by a dysregulated host response to infection [3]. This organ dysfunction is calculated using the Sequential Organ Failure Assessment (SOFA) Score. While the original is quite cumbersome, the qSOFA score is a simplified version easily used at the bedside. While qSOFA has already shown promise in predicting patients with sepsis at risk for increased mortality [4], it remains to be seen whether it is useful at screening patients for sepsis early in the disease process.

Source: http://epmonthly.com/article/sepsis-gets-an-upgrade/

Source: http://epmonthly.com/article/sepsis-gets-an-upgrade/

Comparison of qSOFA score and SIRS criteria as screening mechanisms for emergency department sepsis. Haydar S, Spanier M, Weems P, Wood S, Strout T. Am J Emerg Med. 2017 Jul 6. pii: S0735-6757(17)30509-0.

This study was a retrospective chart review performed at a single academic tertiary care hospital. Its primary objective was to determine both the sensitivity and the diagnostic timeliness of the qSOFA score compared to SIRS criteria in a population of emergency department patients. While not explicitly stated, its secondary objective was to determine the test characteristics (including sensitivity, specificity, negative predictive value, positive predictive value, and the area under the receiver operatic characteristic curve) of qSOFA and SIRS to identify septic patients that would ultimately die in-hospital.

The sample of patients were drawn from a base population that was treated with antibiotics in the ED for a suspected infection, admitted, and ultimately expired or subsequently discharged with a Center for Medicare Services Diagnosis Related Grouping (DRG) for sepsis.

Data were extracted to fulfill the criteria for both qSOFA and SIRS, including respiratory rates, systolic blood pressures, heart rates, white blood cell counts, temperatures, altered mental status (AMS), and the times at which these parameters were documented. Pre-hospital data was excluded. Of note, both physician and nursing documentation were reviewed. Interestingly, laboratory values (i.e., WBC count) were considered to be present at the time of the blood draw, not at the time of result. Data was extracted by a single reviewer for all data points except for the timing of AMS. During a random sampling by two reviewers to determine reliability, the kappa value of 0.4 was deemed inappropriate, prompting a consensus review of all charts to determine initial time of AMS.

Of the 200 sampled patients, one was excluded due to transfer from an outside facility, leaving 199 for analysis (Table 1). Of note, median age was 71 years (range 18-102) and in-hospital mortality was 11.0% (n = 22). The majority of patients were white (97%, n = 194) and Non-Hispanic (100%, n = 200).

Table 1

Table 1

SIRS criteria outperformed the qSOFA score in sensitivity for diagnosing sepsis while in the ED, mean time to diagnosis, and median time to diagnosis (Table 2). The overall sensitivity for qSOFA was quite poor, and in particular, only 36.7% of patients met the AMS requirement. In determining in-hospital mortality, qSOFA had a much higher specificity and positive predictive value, but ultimately the overall performance as evidenced by the AUROC was relatively poor for both SIRS and qSOFA and they did not differ significantly (Figure 1).

Table 2

Table 2

Figure 1

Figure 1

qSOFA came after years of criticism towards the SIRS criteria. It has long been recognized that the SIRS criteria are not specific for infection, and that a variety of conditions including pain, trauma, and nonspecific inflammation can place patients in a SIRS-positive category. A recent study demonstrated that nearly half of hospital ward patients developed positive SIRS criteria at least once during their stay [5]. If not categorized appropriately, this could lead to inappropriate antibiotic utilization and fluid resuscitation.

However, the introduction of qSOFA the Sepsis-3 criteria has its own inherent limitations. Primarily, the focus has shifted towards hypotension and altered mental status as markers of end-organ dysfunction. While these features are intuitively associated with a higher disease burden (i.e., altered patients and those that are hypotensive are clearly sicker than those that are not), it shifts the focus from screening to prognostication. While high qSOFA scores have proven to correlate with in-hospital mortality [4], is this really the most important question for the emergency department physician?

More useful is a tool that screens positive for the largest proportion of potentially infected patients, leaving clinical judgment to further distill appropriate workup and treatment. This paper suggests that SIRS criteria are much better suited for this purpose. Compared to qSOFA, SIRS had a sensitivity for diagnosing sepsis that was over 36% higher, with a reduction in time to diagnosis by approximately one half.

The major strength of this paper is the wide net of inclusion. Essentially, all patients admitted and subsequently diagnosed with sepsis were included and randomly sampled. This group spanned a variety of ages, acuities, and types of infections. This was a strong attempt at making the data as generalizable as possible. However, the population at Tufts Medical Center in Maine, predominately White and Non-Hispanic may limit the external validity to more diverse practice environments.

Timing of diagnosis was another factor fraught with both pros and cons. While not explicitly stated, it seems intuitive that nursing documentation review in addition to physician notes would allow for expedited recognition of vital sign abnormalities and the onset of AMS. However, even with this inclusion, the retrospective determination of AMS onset without objective documentation practices is prone to error. One can imagine that late documentation in a busy emergency department setting may have contributed to the delay in time from arrival to documentation of qSOFA criteria. Furthermore, it seems odd that the laboratory results were considered to be present at the time of blood draw rather than at the time of result availability. Clinically, this is not how we would be able to diagnose sepsis in real-time, and it may have falsely hastened the time from ED arrival to documentation of SIRS criteria in this study.

Lastly, it is important to recognize that the Surviving Sepsis campaign has championed early recognition and treatment as the key principle to reducing morbidity and mortality. This has led many hospital systems to incorporate SIRS criteria into the electronic health record, to flag patients as early as possible for recognition. This is not as feasible with qSOFA, as the component of altered mentation is subject to individual interpretation.

In summary, sepsis is a critical diagnosis that must be made early to improve outcomes. The SEPSIS-3 campaign promotes the use of qSOFA criteria, which are clearly a prognostic marker for increased mortality. However, the SIRS criteria are more useful in screening for sepsis early in the disease process in emergency department patients.


Expert Commentary

As Dr. Ireland’s excellent review of this attempted validation study notes, what was originally heralded as a new paradigm in identifying the warning signs of sepsis (qSOFA) appears to in fact be (significantly) worse than the SIRS criteria we all know and (don’t) love.

As others have noted far more eloquently than I (see: pretty much anything that Josh Farkas of PulmCrit has written on qSOFA), it really shouldn’t be surprising to any of us that qSOFA is less sensitive than SIRS.  By including hypotension and altered mental status (just another word for end-organ dysfunction in the old paradigm), qSOFA is essentially screening for what was formerly known as ‘septic shock.’  Not a surprise that patients with 2 or more of these criteria don’t do well from a mortality standpoint.  This isn’t what I need from a screening test for sepsis in the ED though - I can’t think of the last time I walked into a patient’s room and found them altered, hypotensive and tachypneic and thought to myself, “Hmm, I wonder if they are sick or not?”

While SIRS isn’t perfect, it at least approaches the requisite sensitivity to make it a useful screening test.  It is important to remember that you as the clinician are tasked with providing the specificity - the warning signs of sepsis overlap with many other acute (and less acute) processes.  Patients screening SIRS positive may need aggressive management of conditions other than sepsis, and not every sepsis patient must receive the full sepsis bundle - evaluate each patient fully before initiating protocol-based care.

Ben_Schnapp-12.jpg
 

Benjamin Schnapp, MD

Assistant Residency Program Director, University of Wisconsin-Madison


How to Cite This Post

[Peer-Reviewed, Web Publication]   Ireland A, Iwaki K (2018, September 17). qSOFA SIRS.  [NUEM Blog. Expert Commentary by Schnapp B]. Retrieved from http://www.nuemblog.com/blog/qSOFA


Other Posts You May Enjoy


References:

  1. BoneRC, BalkRA, CerraFB, etal. American College of Chest Physicians/Society of Critical Care Medicine Consensus Conference: definitions for sepsis and organ failure and guidelines for the use of innovative therapies in sepsis. Crit Care Med. 1992;20(6):864-874.

  2. Dellinger RP, Levy MM, Rhodes A, et al. Surviving sepsis campaign: international guidelines for management of severe sepsis and septic shock: 2012. Crit Care Med 2013;41(2):580–637.

  3. Singer M, Deutschman CS, Seymour CW, et al. The third international consensus def- initions for sepsis and septic shock (sepsis-3). JAMA 2016;315(8):801–10.

  4. Freund Y, Lemachatti N, Krastinova E, et al. Prognostic accuracy of sepsis-3 criteria for in-hospital mortality among patients with suspected infection presenting to the emergency department. JAMA 2017;317(3):301–8.

  5. Churpek M.M., Zadravecz F.J., Winslow C., et al: Incidence and prognostic value of the systemic inflammatory response syndrome and organ dysfunctions in ward patients. Am J Respir Crit Care Med 2015; 192: pp. 958-964

  6. Comparison of qSOFA score and SIRS criteria as screening mechanisms for emergency department sepsis. Haydar S, Spanier M, Weems P, Wood S, Strout T. Am J Emerg Med. 2017 Jul 6. pii: S0735-6757(17)30509-0.

Posted on September 17, 2018 and filed under Infectious Disease.

Bruised and broken hearts: diagnosis and management of blunt cardiac injury

Screen Shot 2018-08-08 at 11.37.33 PM.png

Written by:  Paul Trinquero, MD (NUEM PGY-4) Edited by:  Victor Gappmaier, MD (NUEM Alum '18) Expert commentary by: Emily Koeck, MD


Clinical vignette

A 39-year-old male presents as a fall from a two-story window, landing on his left side. He lost consciousness after the fall but is now back to his baseline mental status. Primary survey is intact and his GCS is 15. Secondary survey is notable for a left temporal scalp hematoma and tenderness over his left anterior chest. A CT brain and CT cervical spine are obtained and both are unremarkable. CT chest is notable for two left sided rib fractures and a small underlying pulmonary contusion, without any evidence of hemothorax or pneumothorax. Given his high-risk mechanism for blunt cardiac injury, ECG and troponin are obtained. Troponin is negative but ECG demonstrates a right bundle branch block, with no prior for comparison. The patient remains well appearing and hemodynamically stable. He is asymptomatic other than mild chest wall pain. This typical multiple blunt trauma patient raises some interesting questions:

  • What is blunt cardiac injury and how is it diagnosed?

  • What are the potential complications and how should they be addressed?

  • Do all patients with chest trauma need an ECG? Troponin?

  • What about isolated sternal fractures?

  • What findings merit an emergent echo?

  • When should an otherwise well appearing patient be admitted for observation?

 

Overview of Blunt Cardiac Injury (BCI)

Blunt Cardiac Injury (BCI) encompasses a spectrum of disease caused by significant blunt force transmitted to the heart via a deceleration injury or direct blow to the precordium. Damage is done as a result of direct compression of the heart between the sternum and spine, increased intra-thoracic pressure, deceleration forces (the heart has relatively unrestricted movement in the AP direction so abrupt deceleration can cause a significant impact with the sternum), or direct trauma from fractured ribs1

BCI is an umbrella term that includes a spectrum of potential pathology such as:

  • Comotio Cordis: sudden death due to an ill-timed force during a period of electrical vulnerability

  • Cardiac rupture: traumatic rupture of the myocardium due to compression of a full chamber during early systole or raid deceleration forces shearing the atria from the vena cava or pulmonary veins.[1] Often identified on autopsy due to roughly 90% fatality within minutes

  • Pericardial rupture and cardiac herniation: very rare. Most likely will either result in death before arrival or will not be the direct cause of death.[1]

  • Valvular injury: laceration of aortic cusps can cause aortic insufficiency. Compression of heart during systole can lead to tearing of mitral valves and/or papillary muscle rupture.

  • Septal tear: traumatic ASD or VSD are less common pathological findings identifiable by characteristic loud holosystolic murmurs and echocardiography

  • Coronary artery dissection/thrombosis: rare to occur in isolation

  • Myocardial contusion: edema and necrosis of cardiac myocytes due to blunt traumatic injury

Of the above injuries, most are relatively easy to diagnosis. Comotio cordis, by definition, is not survivable. Cardiac rupture leads to immediate death in most cases, but if a stable hematoma forms, the patient may present alive and in tamponade, which can be identified clinically and with the aid of bedside ultrasound. Isolated pericardial rupture is very rare. It can be associated with cardiac herniation and subsequent impairment in cardiac output, which will manifest with unstable vitals or could be identified on echo. Valvular or septal injuries will often present with heart failure, and most will be associated with a loud, new murmur and/or hemodynamic instability. Coronary artery dissection is exceedingly rare, but diagnosis (ECG, troponin) and treatment (cardiology consultation, PCI) are similar to regular MI and not unfamiliar to the emergency physician. That leaves myocardial contusion, which is the subject of considerable debate and will be discussed in detail below.

There is no clear-cut definition or gold standard diagnosis for myocardial contusion. Pathologically, a cardiac contusion involves edema and necrosis of myocytes as well as patchy areas of hemorrhage, similar to that seen with an MI. Hence, cardiac troponins are very specific for myocardial injury from trauma just as they are for ischemic damage.[2] Serum levels are elevated much more rapidly than after MI, however some sources recommend a 4-6 hr delta troponin depending on time of initial presentation and level of suspicion.[2,3] However, cardiac contusions can occur in the absence of troponin elevation and can be variably diagnosed via TTE, TEE, or ECG. Although frequently encountered in high-risk poly-trauma patients, the vast majority of cardiac contusions tend to improve spontaneously and will heal with scar formation. They are generally well tolerated and may produce only minimal symptoms.[2] Prognosis is excellent both in-hospital and at 3 and 12 month follow up and patients who are initially clinically stable are very unlikely to deteriorate due to cardiac contusion.[4] There are two mechanisms by which blunt cardiac injury can lead to significant morbidity and mortality: significant contractile dysfunction and arrhythmia.

  1. Significant contractile dysfunction is easy to identify by assessing the patient’s vital signs. A hemodynamically stable, asymptomatic patient is unlikely to be suffering from serious traumatic heart failure. Conversely, patients with hemodynamic instability or persistent arrhythmia should have an emergent echocardiogram to assess for a structural abnormality or hemodynamically significant contusion.[3]

  2. Arrhythmia may have a delayed presentation in an otherwise asymptomatic patient. Therefore, “at risk” patients may benefit from a telemetry admission in order to identify and treat expeditiously. Twenty four hours is an appropriate duration for monitoring because evidence suggests that arrhythmia will almost always manifest within the first 24 hours.[2,5] To screen for those at risk, the Eastern Association for the Surgery of Trauma (EAST) guidelines strongly recommend an ECG on all patients with a potential mechanism.[3] Common mechanisms include motor vehicle collisions, falls from height, and crush injuries. In terms of defining a high-risk mechanism, the EAST guidelines are not specific, but many individual institutions specify particular speeds or characteristics of MVC or particular heights of falls that merit screening for BCI.

Of note, while an isolated sternal fracture is clearly indicative of significant force transmitted to the thoracic cavity, is should be thought of as a risk factor for BCI rather than pathognomonic. Only a small percentage of patients with isolated sternal fracture wind up with a cardiac contusion.[6] Hence, patients with sternal fracture should be screened (with an ECG and troponin as discussed above), but should not be immediately labeled with a diagnosis of myocardial contusion or blunt cardiac injury.

Prior guidelines hedged on the utility of a troponin, but the new 2012 EAST guidelines acknowledge several recent studies which have shown that a normal ECG alone may not be sufficient to rule out clinically significant BCI and that the addition of a negative troponin increases negative predictive value to 100%. Patients with a normal ECG and negative troponin can be ruled out for BCI.[3] This guideline is partially based on a prospective study, which evaluated 333 patients with significant thoracic trauma and concluded that patients with a normal ECG and a negative delta troponin (at 0 and 8 hrs) could be safely discharged if they lacked other criteria for admission.[7] Patients with either an ECG abnormality (arrhythmia, ST changes or evidence of ischemia, heart block) or an elevated troponin should be admitted for telemetry monitoring for 24 hours.[3]

Case Resolution

Our patient from above was admitted for 24 hour monitoring given his abnormal initial ECG. In addition to pain control, incentive spirometry, and supportive care for his rib fractures, he was monitored on telemetry given his elevated risk of dysrhythmia from a likely cardiac contusion. Fortunately, he had an uneventful hospital stay, repeat ECG showed resolution of the prior bundle branch block, and he was discharged the following afternoon.

Summary

  • Blunt cardiac injury (BCI) is an umbrella term encompassing a wide spectrum of pathology due to blunt thoracic trauma.

  • Hemodynamically unstable patients should receive an emergent echo. This will help to identify structural abnormalities such as cardiac, septal, or pericardial rupture, valvular disruption, or hemodynamically significant cardiac contusion.

  • At-risk patients should be screened with an ECG and a troponin. If both are normal, then clinically significant BCI is unlikely.

  • Isolated sternal fracture is a risk factor for BCI and should prompt screening with ECG and troponin, but is not pathognomonic and does not mandate additional BCI workup on its own

  • Patients with an abnormal ECG or an elevated troponin should be admitted for telemetry monitoring for 24 hours to ensure timely treatment if the patient develops a dysrhythmia.


Expert Commentary

Excellent overview of a broad and complicated topic; just a few points to clarify/emphasize. As you stated, blunt cardiac injury is truly a spectrum of injuries related to the delivery of significant force to the precordium/chest wall. For the most part, these patients are either stable or nearly dead. The truly serious injuries, such as comotio cordis or free cardiac/pericardial rupture, are generally fatal prior to hospital arrival. Blunt valvular injury tends to be hemodynamically significant and should be suspected in a patient with signs of cardiogenic shock or murmur. While the overall incidence of BCI in the setting of thoracic trauma ranges from 13-76%, it is rare to have serious complications from BCI, and most patients who are alive on arrival to the hospital have minor cardiac injuries. These are usually myocardial contusions or dysrhythmias, and tend to be asymptomatic and self-resolve within 24 hours.

Given the high incidence of BCI and poor sensitivity of physical exam, all patients with an appropriate mechanism should be screened with EKG and troponin. A normal EKG and negative troponin is sufficient to rule OUT blunt cardiac injury. Patients with EKG changes and/or positive troponins should be stratified by hemodynamics and clinical stability. Stable patients should be observed with telemetry for resolution of EKG changes, with serial EKG and troponins depending on the degree of abnormality. Any unstable patient with risk factors for BCI should undergo emergent echocardiography to identify a possible serious injury that would require intervention.

As a final note, the risk factors for BCI are also risk factors for aortic injury, so make sure to evaluate the aorta in unstable or symptomatic patients.

koeck.png
 

Emily Koeck, MD
Surgical Critical Care, Trauma, and Burn Fellow, John H. Stroger, Jr. Hospital of Cook County


How To Cite This Post

[Peer-Reviewed, Web Publication]   Trinquero P, Gappmaier V (2018, September 10). Blunt Cardiac Injury.  [NUEM Blog. Expert Commentary by Koeck E]. Retrieved from http://www.nuemblog.com/blog/BCI


Posts You May Also Enjoy

References:

  1. El-Menyar A, Al Thani H, Zarour A, Latifi R. Understanding traumatic blunt cardiac injury. Ann Card Anaesth. 2012;15(4):287-295.

  2. Sybrandy KC, Cramer MJ, Burgersdijk C. Diagnosing cardiac contusion: old wisdom and new insights. Heart. 2003;89(5):485-489.

  3. Clancy K, Velopulos C, Bilaniuk JW, et al. Screening for blunt cardiac injury: an Eastern Association for the Surgery of Trauma practice management guideline. J Trauma Acute Care Surg. 2012;73(5 Suppl 4):S301-306.

  4. Lindstaedt M, Germing A, Lawo T, et al. Acute and long-term clinical significance of myocardial contusion following blunt thoracic trauma: results of a prospective study. J Trauma. 2002;52(3):479-485.

  5. Fabian TC, Cicala RS, Croce MA, et al. A prospective evaluation of myocardial contusion: correlation of significant arrhythmias and cardiac output with CPK-MB measurements. J Trauma. 1991;31(5):653-659; discussion 659-660.

  6. Athanassiadi K, Gerazounis M, Moustardas M, Metaxas E. Sternal fractures: retrospective analysis of 100 cases. World J Surg. 2002;26(10):1243-1246.

  7. Velmahos GC, Karaiskakis M, Salim A, et al. Normal electrocardiography and serum troponin I levels preclude the presence of clinically significant blunt cardiac injury. J Trauma. 2003;54(1):45-50; discussion 50-41.

 

Posted on September 10, 2018 and filed under Trauma.

Uvular Edema

Screen Shot 2018-08-08 at 10.58.39 PM.png

Written by:  Gabby Ahlzadeh, MD (NUEM PGY-4) Edited by:  Rachel Haney, MD (NUEM Alum '17) Expert commentary by: Gentry Wilkerson, MD (University of Maryland)


It’s a busy overnight and the nurses are speedily wheeling a gentleman from triage to the resuscitation bay. “It’s an allergic reaction, come quick!” You take a look at the patient: no respiratory distress, no lip swelling, no facial swelling, no hives, seems pretty comfortable. You ask a question and the patient has a hot potato muffled voice and seems like he has something stuck in the back of his throat. No wheezing, satting well on room air, no trismus, the patient opens wide and all you see is uvula: edematous, enlarged, looks like a very large grape. The patient is tolerating his secretions well otherwise. Is this an allergic reaction w/no other systemic symptoms and no clear precipitant? Is this uvulitis?  Is there a peritonsillar abscess? Does this person need to be intubated?

3 recent cases demonstrate the wide variety of ways that uvular edema can present:

Case 1

A male took a few shots of aged whiskey prior to sleeping then woke up a few hours later with a swollen uvula, no other allergic symptoms. He improved with IM epinephrine x3, NP scope showed normal appearing epiglottis and vocal cords; he was admitted to the ICU for close monitoring and had resolution of symptoms within 24 hours.   

Case 2

An older gentleman on an ACE-inhibitor  presented with 2 weeks of sore throat.  He had isolated uvular edema years ago after drinking vodka. He avoided vodka since then, but drank tequila the night prior to presentation and woke up with a grapelike uvula. We treated him for infectious and allergic causes and he was also admitted for respiratory monitoring.

Case 3

A man in town for business who had previously had uvular edema and was usually able to manage his symptoms at home with Benadryl and IM epi, but had forgotten his epi pen in a different suitcase. He improved after steroids, epinephrine, antihistamines. After a period of observation he wanted to leave. We discharged him with steroids and an epi pen and recommended he follow up with an allergist.

Differential for uvular edema:

  • Epiglottitis – fever, drooling, anxiety, airway obstruction, think of kids, get lateral neck films and be careful examining the kids- they need to be kept calm as agitation can worsen airway obstruction!

  • Retropharyngeal abscesses or cellulitis – midline or unilateral swelling of posterior oropharynx, stridor tachypnea, won’t extend the neck, fever, intense pain with swallowing, think CT scan

  • Peritonsillar abscess – tonsillar swelling, deviated uvula, fever, sore throat, trismus, hot potato voice (only 1 case of uvulitis w/PTA has been reported).

  • Angioedema – AKA Quincke’s edema, foreign body sensation, grape like in appearance, uvular hydrops, maybe eosinophilia on CBC, similar occurrence in the past

  • Viral exanthem – vesicular lesions

  • Severe pharyngitis – pharyngeal edema sore throat, palatal petechial, tonsillar enlargement, exudates

  • Mechanical Trauma – ulceration of the uvula, compression from LMA or ETT, recent ENT procedure

  • Drugs – inhaled cocaine, cannabis, herbal medicines like the juice of the squirting cucumber or Ecballium elaterium, used as a homeopathic remedy for sinusitis. Can be the exposure itself or as a result of thermal injury.

  • Hereditary angioneurotic edema (HANE) – autosomal dominant genetic form, lack of C1 esterase inhibitor protein, think about w history of repetitive episodes of uvular edema, family history, confirmed with blood samples showing low C1 esterase levels. If suspected, can use bradykinin receptor antagonist (Icatibant) or complement C1 inhibitor concentrate (Berinert, Cinryze).

 Literature:

  • Most cases are case reports and involve exposure to drugs, inhaled substances.

  • One Spanish study from 2010 found that of 58 patients who presented with uvular edema, 75.9% presented with isolated uvular edema; 55.1% were idiopathic with predisposing factors of being overweight, longer uvula, GERD, and having a tendency to snore. Recurrent episodes were more common in the idiopathic group as well

  • Snoring has been found to precipitate uvular angioedema in patients taking ACE inhibitors

  • While it seems there is no specific data about management of idiopathic cases, most are treated as caused by an allergic reaction

  • There is no clear cut recommendation about whether these individuals should be admitted for respiratory monitoring or whether isolated uvular edema can truly obstruct the airway

Management:

  • If infectious etiology (fever, pain), treat as such and the uvular edema is likely reactive. Think group A streptococci, Haemophilus influenzae, Streptococcus pneumoniae

  • Keep patient in upright position to minimize airway obstruction

  • Rule out epiglottitis ASAP with lateral neck films or NP scope if patient can tolerate

  • Allergy cocktail: antihistamine, epinephrine, steroids, H2 blocker; discharge with Epi pen

  • Uvula irritates posterior OP causing nausea, so Zofran can help

  • Topical epinephrine or inhaled nebulized epinephrine for vasoconstriction to decrease edema

  • Needle decompression of uvula has been done in the past with only anecdotal evidence

  • Rhinolaryngoscopy to rule out epiglottitis if patient is not improving over time; might be a good idea to have the ETT loaded onto the scope just in case there is cord edema or acute airway obstruction during the procedure.

  • If intubation is needed, the uvula will certainly be in the way so reach for the fiberoptic scope or just clamp the uvula and pull it to the side.

  • Consider observation for airway monitoring


Expert Commentary

This blog post is an interesting discussion about the patient presenting with isolated uvular swelling. The uvula is the fleshy structure that hangs from the soft palate in the posterior pharynx. It is composed of glandular and connective tissue with interspersed muscle fibers. Seromucous glands within the uvula produce much of the total volume of saliva.  Patients presenting with uvular complications will often have some combination of dysphonia, dysphagia, and dyspnea.

The underlying cause of uvular swelling can be due to trauma, infection, inflammation, and angioedema due to allergic reactions and non-allergic mechanisms. Performing a comprehensive history and physical will often help provide guidance about the cause of the problem. However, up to half of all cases of uvular swelling will have no identifiable cause. Trauma to the uvula can occur as a result of direct physical contact, thermal or cold exposure, and vibration (as with snoring). Uvular hematoma has been seen in cases of thrombolytic administration. Isolated infection of the uvula is very uncommon. It usually occurs in the setting of more widespread infection as with pharyngitis, tonsillitis, or epiglottitis. Pathogens responsible include Haemophilus and Streptococcus species as well as due to candidal infections.

Angioedema is a term that describes the physical exam finding of transient, nonpitting swelling of subcutaneous tissue or of the submucosal layer of the respiratory or gastrointestinal tracts. Isolated uvular angioedema has been called Quinke’s edema in recognition of Heinrich Quinke’s contribution to the understanding of angioedema. Most forms of angioedema result from increased levels of either histamine or bradykinin. Histaminergic angioedema is typically allergic or immunologic. Bradykinin-mediated forms of angioedema include hereditary angioedema, acquired angioedema, ACE-inhibitor induced angioedema. The term “angioneurotic edema” is archaic and refers to the earlier belief that angioedema was the result of neurologic or psychiatric disturbances. Differentiating between histaminergic and bradykinin-mediated forms of angioedema can be difficult due to the lack of available testing in the Emergency Department. Histaminergic forms may be associated antecedent exposure to a possible allergen and subsequent development of urticaria and pruritus whereas bradykinin-mediated forms are not.

Regardless of the cause of uvular swelling, the most important component of treatment is airway management. There is no definitive point at which it can be clearly determined that a definitive airway needs to be obtained. The decision must be made based on a combination of factors including rapidity of disease progression, anatomic considerations that may make intubation more difficult and equipment available to the clinician. Once the decision to intubate is made, it should be performed by the most experienced provider with anticipation of a difficult airway. Many experts suggest preparing a “double set-up” where the neck is prepped for a cricothyroidotomy in the event of a failed airway.

Performance of nasopharyngoscopy is somewhat controversial but I believe that it is of great importance to fully understand the extent of disease. The clinician should be aware that any physical manipulation of the airway may result in worsening of the swelling and therefore he or she should be prepared to immediately secure the airway.

Frequently, cases are treated with a shotgun approach where patients are treated with epinephrine, steroids and antihistamines. In cases of hereditary angioedema there are now a number of FDA-approved medications that act by replacing C1 esterase inhibitor (C1-INH), inhibiting kallikrein mediated breakdown of high molecular weight kininogen (HMWK) into bradykinin or inhibiting the bradykinin B2 receptor. Despite case reports and case series none of these have been shown to be effective in ACE inhibitor induced angioedema. Fresh frozen plasma contains both C1-INH and angiotensin converting enzyme (ACE, also known as kininase II), which may help to reduce the swelling associated with bradykinin-mediated forms of angioedema. FFP also contains HMWK and kallikrein, which may result in increased formation of bradykinin. Any concern for infection should prompt the clinician to provide appropriate antimicrobial or antifungal coverage.

Any patient that has swelling involving the airway will need close monitoring until the swelling resolves. Most will require admission to an intensive care unit where prompt airway management can occur in the event of clinical deterioration. In 1999, Ishoo et al performed a single-center, retrospective review of patients admitted over an eleven-year period with angioedema due to all causes.  They found the following factors were associated with an increased risk of need for definitive airway: voice change, hoarseness, stridor and dyspnea. Patients were categorized by the location of angioedema into 4 non-continuous stages. Application of this categorization has limitations as there have been numerous advances in management in the two decades since this was published.

wilkerson.png
 

Gentry WIlkerson, MD

Assistant Residency Program Director, University of Maryland Medical Center


How to Cite this Post

[Peer-Reviewed, Web Publication]   Ahlzadeh G, Haney R (2018, September 3). Uvular Edema.  [NUEM Blog. Expert Commentary by Wilkerson G]. Retrieved from http://www.nuemblog.com/blog/uvular-edema


Other Posts You May Enjoy


References:

  1. Alcoceba E, Gonzalez M, Gaig P, et al. Edema of the Uvula: Etiology, Risk Factors, Diagnosis and Treatment. J Investg Allergol Clin Immunol. 2010;20(1):80-3. http://www.jiaci.org/issues/vol20issue1/12.pdf

  2. Evans TC, Roberge RJ: Quincke's disease of the uvula. Am J Emerg Med 1987;5:211-216.

  3. Goldberg R, Lawton R, Newton E et al. Evaluation and management of acute uvular edema. Ann Emerg Med.1993;22:251-255

  4. Kuo DC, Barish RA. Isolated uvular angioedema associated with ace inhibitor use. J Emerg Med 1995;13:327–30

  5. Rasmussen E, Mey K, Bygum A. Isolated oedema of the uvula induced by intense snoring and ACE inhibitor. BMJ Case Reports, vol 2014; 2014.

  6. Roberts J. Acute angioedema of the Uvula. Emergency Medicine News. 2001;23(7):7-12.

  7. Welling A. Enlarged uvula (Quincke’s Oedema) – A side effect of inhale cocaine? – a case study and review of the literature. International Emergency Nursing. 2008;16(3):207-10.

 

Posted on September 3, 2018 and filed under ENT.

Ultrasound-guided Peripheral IJ Catheter Placement

Screen Shot 2018-08-08 at 10.14.23 PM.png

Written by:  Samantha Knopp, MD (NUEM PGY-3) Edited by:  Andrew Ketterer, MD (NUEM Alum '17) Expert commentary by: John Bailitz, MD


We’re all familiar with the “difficult access” patient: the nurses have tried all possible traditional peripheral routes, both ultrasound-guided and not, the resident has been in with the ultrasound and had no better luck, the EJ blew a few minutes after it was placed. The choices you seem to be left with are intraosseous access (certainly useful in an actual emergent situation, although having a spike drilled into their long bones is not something that most awake and alert patients are thrilled about) or gain central access via a central venous catheter (again, useful and appropriate in some circumstances, but poses increased risk for complications).

Fortunately, there is a third option! The ultrasound-guided catheterization of the IJ with a peripheral IV, a technique first described in the literature in 2009 [1], has been shown to be a safe and efficacious means of access when all else fails. [2,3]

 

What is it?

Ultrasound-guided placement of a standard single-lumen angiocatheter into the internal jugular vein.

When is it useful?

In patients who require an IV, and no suitable extremity or external jugular veins can be reliably accessed, assuming that:

  1. the patient is not unstable requiring emergent resuscitation (in which case an IO is preferable), and

  2. the patient does not require central venous access

How to do it

The perennially creative people over at EM:RAP have an excellent video demonstration of the peripheral IJ:

  1. What you’ll need:

    • Ultrasound machine with linear transducer

    • Sterile ultrasound gel

    • Chlorhexidine

    • Tegaderm x 2 (or other bio-occlusive dressing; 1 for dressing, 1 to cover ultrasound probe)

    • Single lumen angiocatheter (various studies have used varying sizes: 18-20 gauge, 4.8cm-6.35cm)

    • Loop catheter extension

    • Saline flush

  2. How you'll do it:

    • Place patient is supine position (can also use Trendelenburg)

    • Use ultrasound to visualize IJ

    • Prep the area with chlorhexidine and drape the patient (limited draping, see video)

    • Cover probe with Tegaderm or sterile probe cover

    • Visualize vessel once again, using sterile jelly and have the patient perform Valsalva maneuver

    • Puncture the skin at a 45-degree angle and advance needle into the IJ lumen

    • Once flash is observed, advance the catheter into the lumen and withdraw the needle

    • Connect the loop catheter extension, ensure that blood draws back, then flush the tubing and apply dressing

 

The Evidence

Accessing the IJ with a peripheral venous catheter was first described in a 2009 letter to the editor in the Journal of Emergency medicine.[1] Only a few studies were subsequently published between 2009 and 2016 regarding the procedure’s technique, its safety, or its efficacy. The few small case series that were published studied 37 patients in total; in all series, the procedure was noted to have a high success rate and on average took significantly less time than placing a central IJ catheter.[5,6,7] The past year has seen two additional prospective studies evaluating both the efficacy and the safety of the peripheral IJ, enrolling a total of 107 patients.[2,3] The first study noted no complications at 1 and 6 weeks associated with US-guided peripheral IJ catheterization.[2] The second, a multicenter study, noted an 88% success rate and a 14% complication rate (the only complication being lost patency—of note, it is unclear whether or not this was considered a complication in the first study).[3] In all studies, the time to insert the peripheral IJ was approximately 5 minutes or less. While the body of literature thus far is still relatively small, it would seem to suggest that the use of a peripheral IJ is a safe and suitable alternative in appropriately selected patients who have no other feasible routes of vascular access, and in whom the insertion of an IO or central line is otherwise unnecessary.

The Takeaway

  1. The placement of a peripheral IV into the internal jugular vein under ultrasound guidance has been described as efficacious and safe.

  2. On average, it is not a time-consuming procedure. This is operator-dependent, but it takes significantly less time than placing a central venous catheter in most cases and is associated with fewer complications.


Expert Commentary

The rare but classic case remains the difficult vascular access patient with severe shortness of breath. Using either the long angiocatheter in the central line kit, and today a long peripheral intravenous catheter, an experienced clinician sonographer may be able to insert the catheter with the patient nearly upright. In such patients, either an infraclavicular subclavian or supraclavicular subclavian central line approach may result in a pneumothorax, quickly turning a bad situation into a nightmare for everyone. Instead, quickly placing a simple long peripheral catheter into the IJ using US guidance immediately establishes the vascular access needed to administer life saving medications. When the patient is stabilized, the traditional central line may then be placed if still required.

Necessity breeds invention! So it is exciting for new and experienced clinicians alike to now be able simply use the long peripheral IV catheter in both stable patients not needing central access, and the rare unstable patients who must remain upright, and only opening an expensive central line kit when needed.

John-Bailitz.png
 

John Bailitz, MD

Associate Professor of Emergency Medicine


How you cite this post

[Peer-Reviewed, Web Publication]   Knopp S, Ketterer A (2018, August 27). Ultrasound-guided peripheral IJ catheter placement.  [NUEM Blog. Expert Commentary by Bailitz J]. Retrieved from http://www.nuemblog.com/blog/peripheral-IJ


Posts you may also enjoy


References

  1. Moayedi, Siamak, “Ultrasound-Guided Venous Access with a Single Lumen Catheter into the Internal Jugular Vein.” The Journal of Emergency Medicine. 2009;37(4):419

  2. Kiefer D, Keller SM, Weekes A. “Prospective evaluation of ultrasound-guided short catheter placement in internal jugular veins of difficult venous access patients.” Am J Emerg Med. 2016 Mar;34(3):578-81

  3. Moayedi S, Witting M, Pirotte M. “Safety and Efficacy of the “Easy Internal Jugular (IJ)”: An Approach to Difficult Intravenous Access” J Emerg Med. 2016Dec;51(6):636-42

  4. EM:RAP <https://www.youtube.com/watch?v=FjSmbUWXznY>

  5. Butterfield M, Abdelghani R, Mohamad M, Limsuwat C, Kheir F. “Using Ultrasound-Guided Peripheral Catheterization of the Internal Jugular Vein in Patients With Difficult Peripheral Access.” Am J Ther. 2015 Oct 8.

  6. Teismann N, Knight R, Rehrer M, Shah S, Nagdev A, Stone M. “The Ultrasound-guided “Peripheral IJ”: Internal Jugular Vein Catheterization using a Standard Intravenous Catheter” J Emerg Med. 2013Jan;44(1):150-54

  7. Zwank, Michael. “Ultrasound-guided catheter-over-needle internal jugular vein catheterization.” Am J Emerg Med. 2012Feb;30(2):372-73

Posted on August 27, 2018 and filed under Procedures.

Toxic Flames

Screen Shot 2018-08-08 at 9.57.05 PM.png

Written by:  Vidya Eswaran, MD (NUEM PGY-3) Edited by:  Jonathan Andereck, MD (NUEM PGY-4) Expert commentary by: Matt Zuckerman, MD (University of Colorado)



Expert Commentary

Thank you, this highlights an important aspect of treating victims of smoke inhalation.

In terms of the physiology of CO I like to think of it as an acquired hemoglobinopathy at low doses, thus patients with premorbid cardiopulmonary disease may be affected at lower doses. A fair amount has been written about how absolute levels correlate poorly with clinical effects. The idea of levels correlating to symptoms seem to originate from a Bureau of Mines publication from 1923 that won’t disappear. I would suggest having a low threshold for testing anyone who might have exposure; the failure for CO is in not testing.

Additionally, cherry lips are rarely found in living patients (more commonly on autopsy event at levels below 50%) so are rarely clinically useful (J Forensic Sci. 1995 Jul;40(4):596-8).

The “consider” HBO recommendation for COHb levels >25% is very controversial and the literature is limited by heterogeneity in patients and treatment protocols. Some would argue against hyperbaric for most patients or even consider HBO for patients at lower levels. Consultation with toxicologists and hyperbaricists is likely to be helpful.

Lactic acidosis is key to cyanide poisoning. Most use a combination of smoke exposure with an elevated lactate (>10 mmol/L) to be highly suggestive of CN toxicity and an indication for empiric treatment. CN levels are rarely helpful and rarely ordered. The description of cyanide symptoms “progressing” is a bit of a misnomer as cyanide is initially rapid onset, without evolving symptoms; indeed knockdown is a common presenting symptom. Hydroxocobalamin is preferred to the antidote kit, and amyl nitrate is omitted if sodium nitrite is given. The transient hypertension associated with hydroxocobalamin is often therapeutic given the incidence of hypotension, and its important to be aware that this will discolor serum and tears and urine.

Picture1.png
 

Matthew Zuckerman, MD

Assistant Professor of Emergency Medicine, University of Colorado School of Medicine


How to cite this post

[Peer-Reviewed, Web Publication]   Eswaran V, Andereck J (2018, August 20). Toxic Flames.  [NUEM Blog. Expert Commentary by Zuckerman M]. Retrieved from http://www.nuemblog.com/blog/toxic-flames


Posts you may also enjoy

Posted on August 20, 2018 and filed under Toxicology.

Pulmonary Hypertension in the ED

Screen Shot 2018-08-07 at 2.53.55 AM.png

Written by:  Kaitlin Ray, MD (NUEM PGY-3) Edited by:  Mitali Parmar, MD (NUEM alum '18) Expert commentary by: Colin McCloskey, MD (NUEM alum '16)


ED Management of Pulmonary Hypertension

Pulmonary hypertension (PH) is rare disease associated with high morbidity and mortality. Given the non-specific nature of pulmonary hypertension in its early stages, it is often only diagnosed once patients have reached an advanced stage of disease [1]. Given the low physiologic reserve of these patients, any superimposed illness, change in volume status, tachyarrhythmias, or changes in oxygenation or ventilation can tip the patient’s homeostatic balance and precipitate a life threatening situation [2]. Presently, no set guidelines exist regarding the management of critically ill patients with pulmonary hypertension in the emergency department (ED). As emergency physicians, we must have a sound understanding of pulmonary hypertension because although a rare disease, management is based on efficiently and effectively addressing and optimizing the underlying pathophysiology [3]. Below is a quick review of the etiology and pathophysiology of pulmonary hypertension, followed by management goals in the ED with regards to optimizing oxygenation, ventilation and volume status, as well as guidelines for resuscitative efforts.

 

Definition/Etiology of PH:

The pulmonary vascular system is a high flow, low resistance circuit. Pulmonary hypertension is defined as mean pulmonary arterial pressure > 25 mmHg at rest (>30 mmHg during exertion) as diagnosed by right heart catheterization. Note that an RV systolic pressure >35mmHg on echo is highly suggestive of PH, however is not diagnostic [1].

Understanding the etiology of PH is critical as it guides treatment. For example, PH secondary to COPD may be addressed by treating the COPD itself. The WHO has classified PH into five groups based on etiology as below [1]:

  • Group 1: Pulmonary arterial hypertension (PAH): may be idiopathic or inherited; secondary to connective tissue disease (scleroderma), HIV, sickle cell disease, etc

  • Group 2: Pulmonary venous hypertension due to left heart disease:

    • Most common cause of PH

    • 2/2 cardiomyopathy, diastolic dysfunction, MS, MR, AS, AR

  • Group 3: Chronic hypoxemic lung disease: COPD, ILD, OSA

  • Group 4: Thromboembolic disease

  • Group 5: Miscellaneous: systemic disorders (sarcoidosis, neurofibromatosis), lymphatic obstruction, hematologic disorders (myeloproliferative)

 

Pathophysiology:

The exact pathophysiology of PH is unknown; however PAH is thought to be secondary to endothelial dysfunction with an imbalance between endogenous vasodilators (ie prostacyclin) and vasoconstrictors (endothelin-1)—resulting in a net effect of vasoconstriction and thrombus formation, leading to elevated pulmonary vascular resistance and impaired blood flow [1].

When pulmonary vascular resistance (PVR) is high, the right ventricle (RV) dilates in order to maintain preload and stroke volume [3]. Over time, displacement of the RV leads to increased ventricular wall tension and inhibits left ventricular filling, causing decreased cardiac output and systemic perfusion [1]. Additionally, the RV is typically perfused during both systole and diastole because of low RV wall tension [2]. However in chronic PH, RV remodeling leads to elevated transmural pressures, thus impairing RCA perfusion such that it occurs only in diastole. This ultimately leads to RV ischemia and potentially RV failure3.

 

The Presentation:

Patients with PH often present with fairly non-specific complaints, with dyspnea (both at rest and with exertion) being the most common. Other complaints include chest pain, fatigue, presyncope/syncope, and exertional lightheadedness. While physical exam findings may be normal early in the course of disease, in more advanced disease assess for signs of RV failure including tricuspid regurgitation, JVD, hepatomegaly, ascites, lower extremity edema, and increased P2 on auscultation.

 

The Workup:

Workup of suspected or confirmed pulmonary hypertension will vary based on the patient, however below are a few easily obtained diagnostics that may assist in your assessment and treatment of the patient.

  • EKG:

    • Most common EKG finding in PH: Right axis deviation [1]

    • Most common dysrhythmias: Atrial fibrillation, atrial flutter, and AVNRT1

    • Look for RVH, RBBB, rsR’ in V1, qR in V1, large inferior P waves, ST depression or TWI in V1 or inferior leads (indicating R heart strain) [3]

  • Chest XR:

    • Evaluate for enlarged RA, RV, and hilar pulmonary arteries

    • Depending on etiology of PH—pulmonary edema, hyperinflation, ILD [1]

  • Bedside echo:

    • Assess the RV—evaluate for RA/RV dilation, RV:LV ratio > 1.0 (normal <0.6) on apical 4-chamber view

    • “D” sign indicating RV pressure overload

    • RV free wall thickening (vs. RV strain due to PE which would result in a thin free wall) [3]

  • Labs:

    • Troponin: if elevated, concern for ischemia due to poor RCA perfusion, associated with increased morbidity and mortality [1]

    • BNP: typically does not impact ED management however can reflect degree of myocardial stretch; can be useful if you also have a baseline for comparison

 

The Goals:

  • Avoid hypoxemia

    • Goal SpO2 > 90% [1]

    • Provide supplemental oxygen as needed

    • Hypoxemia/hypercapnea --> vasoconstriction in lungs --> worsening pulmonary vascular resistance [3]

  • Avoid intubation…

    • Increased risk of rapid cardiovascular collapse with intubation [1]

    • Increased intrathoracic pressure from positive pressure ventilation --> decreases preload --> worsening cardiac output.

    • Avoid NIPPV in the setting of hypotension as this will also increase intrathoracic pressure and therefore decrease preload [3]

  • …but if you must intubate:

    • Etomidate for induction: minimal effects of systemic vascular resistances, pulmonary vascular resistance, and cardiac contractility [3]

    • Use lung protective settings (TV of 6ml/kg ideal body weight, lowest PEEP to maintain O2 >90%)

    • Monitor serial plateau pressures (<30cm H20)

    • Avoid hypercapnea: adjust respiratory rate as needed [1]

      • Recall that hypercapnea increases pulmonary vascular resistance, pulmonary artery pressure, and RV strain

  • Optimize intravascular volume:

    • Assess volume status: Physical exam is often unreliable in patients with PH; trends in CVP may be useful so consider early placement of a central line [1]

    • If clearly hypovolemic: give serial 250cc boluses with close monitoring. Start low and go slow! [2]

    • If clearly hypervolemic: cautiously diurese (furosemide, bumetanide) and titrate to patient’s response

      • Hypervolemia --> RV dilation --> displaced intraventricular septum --> decreased LV volume --> decreased cardiac output --> decreased systemic perfusion [2]

    • Pulmonary artery catheters: most reliable method to manage fluid balance in an ICU but has not been shown to improve mortality [2]

    • If patient proves refractory to volume management:

      • Consider RV assist device

      • Consider inhaled NO

      • Consider VA ECMO (biventricular support and respiratory support [2]

  • Augment RV function:

    • Dobutamine: drug of choice!

      • Beta-2 mediated systemic vasodilation

      • Increases contractility, reduces pulmonary and systemic vascular resistance [3]

      • Avoid > 10 micrograms/kg/min --> may increase PVR, cause tachydysrhythmias, or hypotension! [1]

      • If hypotensive on dobutamine --> start norepinephrine! [3]

    • Milrinone: 2nd line

      • PDE-3 inhibitor --> reduces PVR to augment RV function

      • Avoid high doses --> may cause hypotension [1]

      • If hypotensive on milrinone --> start norepinephrine! [3]

  • Maintain RCA perfusion:

    • Norepinephrine: drug of choice!

      • Alpha-1/alpha-2 properties increase systemic vascular resistance --> augments RV function and CO

      • Reduces 28-day mortality from cardiogenic shock [3]

      • Avoid dopamine and phenylephrine due to increased risk of tachydysrhythmias and elevation in PVR and pulmonary artery pressure [1]

  • Rate control dysrhythmias:

    • Most common arrhythmias = atrial fibrillation/atrial flutter

    • If uncontrolled can precipitate acute decompensation

    • Treat aggressively: if unstable, low threshold to cardiovert

    • Caution with beta-blockers/calcium-channel blockers: impair contractility and may cause cardiogenic shock [3]

  • Decreased RV afterload:

    • Pulmonary vasodilators: decreasing pulmonary arterial pressure will decrease RV afterload [3]

    • Most commonly used pulmonary vasodilators [2]:

      • Prostanoids: rarely started in ED, often given via ongoing infusion

      • Endothelin receptor antagonists: PO, not typically used in acutely ill

      • PDE-5 inhibitors: PO, not typically used in acutely ill

  • Troubleshoot: Avoid disruptions in medication!

    • If patient prescribed PO medication but is unable to receive it in the ED, start an inhaled or IV therapy while consulting with patient’s PH specialist [2]

    • If patient has continuous prostanoid infusion via central venous catheter with a portable infusion pump, do not discontinue the pump!

      • If pump is malfunctioning, consider this a life-threatening emergency! Patient is at increased risk of RV failure, rebound pulmonary hypertension and death.

      • Place IV line and reinitiate the pump while simultaneously calling a PH specialist

      • Do NOT interrupt the infusion for any circumstance

      • Do NOT turn off the pump

      • Do NOT prime or flush the IV line—a bolus with too much medication can be just as dangerous as lack of medication

      • Do NOT infuse other medications where the PH medication is infusing (obtain 2nd peripheral IV if needed) [4]

    • If patient presents with adverse effects associated with medication due to systemic vasodilation (ie flushing, headache, diarrhea, jaw discomfort), do NOT stop or decrease dose of medication! [2]

 

The Disposition:

The majority of these patients will be admitted to the hospital for continued management. For those in acute RV failure, admission to the ICU is more appropriate. If patient is well appearing and you are considering discharge, obtain a walking O2 saturation. If patient desats, they should likely be admitted.

 

The Recap:

Pulmonary hypertension can be difficult to manage as these patients have little physiologic reserve and volume status can be difficult to assess. Realizing that there are no specific guidelines for ED management in critically ill patients with PH, we must guide our treatment based on the pathophysiology of the disease. Keeping in mind these basic principles as listed below, we can more efficiently and effectively treat patients with PH.

  • Treat the underlying cause if able!

  • Avoid hypoxemia

  • Avoid intubation, but if you must, use etomidate for induction and place vent on lung protective settings

  • Optimize intravascular volume: Give small 250cc boluses if hypovolemic and cautiously diurese if hypervolemic—constantly titrate your efforts towards the patient’s hemodynamic response

  • Augment RV function: 1st line = dobutamine, 2nd line = milrinone

  • Maintain RCA perfusion: 1st line = norepinephrine

  • Rate control dysrhythmias: low threshold to cardiovert patients in uncontrolled atrial fibrillation or flutter

  • Decrease RV afterload: pulmonary vasodilators

  • Avoid any kind of disruption in medication delivery (whether PO or via continuous infusion via central venous catheter with portable pump)


Expert Commentary

This is an excellent overview of pulmonary hypertension for the emergency physician. Several points of emphasis include:

  1. Pulmonary hypertension, and its therapeutic considerations, is not as rare as it may seem. Although WHO class 1 pulmonary arterial hypertension (PAH) has an incidence of 15 per 1 million patients, pathologies featuring right ventricular (RV) dysfunction are common. 10-30% of patients with COPD have elevated pulmonary artery pressures [1]. The prevalence of echocardiographic right ventricular dysfunction in ARDS is 22-50% [2]. Sepsis can cause right ventricular dysfunction itself [3], and infection is the most common cause of acute RV failure in patients with PAH [4]. Thus, patients with right heart dysfunction, either from primary PAH as described above, or secondary to a concomitant pathology are omnipresent in the emergency department.

  2. Echocardiography is essential in evaluating these patients: For one, it can rule out physiologic mimics of right heart dysfunction, such as cardiac tamponade. It can also reliably show systolic dysfunction of RV, with use of the tricuspid angular plane systolic excursion (TAPSE). A TAPSE < 15 mm yielded high specificity to distinguish abnormal from normal RV EF [5,6]. Further, if there is a question on if right heart dysfunction is acute or chronic, measurement of the RV free wall (normal 3-5 mm) correlate with chronicity of elevated right sided pressures [7].

  3. In addition to BNP and troponin, abnormal liver function in conjunction with concern for RV failure has a negative prognostic implication [8,9]. LFT elevation with hypoxia and a clean chest x-ray should prime concern for RV pathology.

  4. Volume status: As you cogently point out, volume status is an essential consideration in these patients. Both high and low filling pressures may result in reduced cardiac output [10]. My approach in the patient with acute heart failure is to perform a passive leg raise or mini bolus of fluid, and do an ultrasound or other assessment of cardiac output. If responsive, then repeat with gentle fluid loading. More often, especially in chronic pulmonary hypertensive patients, diuresis is more often required.

  5. Inotropes: Dobutamine, milrinone and digoxin are all acceptable. Milrinone may be novel to most EPs; it is a PDE 3 inhibitor given as a loading bolus followed by an infusion. Evidence exists that it lowers pulmonary vascular resistance to a greater extent than dobutamine [11,12]. Similar to dobutamine, it can cause systemic hypotension, and may require a vasopressor or inopressor. An oft forgotten inotropic agent that is useful in these patients is digoxin [13]. It offers RV systolic support with benign effects on heart rate. A digoxin load (500 mcg q2 hrs up to 1.5 mg) can be effective in the tachycardic patient who needs right sided inotropic support.

  6. If systolic blood pressure requires augmentation, norepinephrine is preferred [14]. RV mechanics improved with NE infusion vs fluid challenge in basic science studies [15], and familiarity of use to EP makes it attractive. Vasopressin at low doses (<0.03 units/min) causes pulmonary vasodilation [16], though at higher doses can increase PVR and cause coronary vasoconstriction. Thus, in a patient in which arrhythmia is a concern this agent is a reasonable choice.

  7. With obvious exception of patients dependent on vasodilator medications via pump, inhaled pulmonary vasodilators are preferred to systemic vasodilators. Pulmonary vasodilators, such as inhaled NO or iloprost, can improve oxygenation in the short term, though are not associated with improvement in mortality [17]. They are preferred to IV vasodilators which can cause systemic hypotension and worsen shunt. Nicely, iNO can be administered via BiPAP or heated high flow nasal cannula.

  8. Intubating these patients is dangerous [18]. RV failure patients should not be intubated solely due to signs of shock, as this can be reversed with aforementioned strategies. Non-invasive forms of ventilation CPAP/BiPAP/HHFNC are all excellent options, perhaps with concomitant inhaled pulmonary vasodilators. Hemodynamic optimization prior to intubation attempt (Resuscitate before intubate), induction with cardiac stable medications (etomidate, ketamine), and lung protective ventilation strategies that allow the least PEEP to ensure adequate oxygenation. However, unlike the ARDSnet protocol, permissive hypercapnia should not be tolerated.

 

  1. Elwing J, Panos RJ. Pulmonary hypertension associated with COPD. Int J Chron Obstruct Pulmon Dis. 2008;3(1):55-70.

  2. Zochios V, Parhar K, Tunnicliffe W, Roscoe A, Gao F. The right ventricle in ARDS. Chest. 2017;152(1):181-193.

  3. Vallabhajosyula S, Kashyap R, Geske J, Kumar M, Kashani K, Jentzer J. 28: Right ventricular dysfunction in sepsis and septic shock an eight-year analysis. Crit Care Med. 2016;44(12):93.

  4. Hoeper MM, Granton J. Intensive care unit management of patients with severe pulmonary hypertension and right heart failure. American journal of respiratory and critical care medicine. 2011;184(10):1114-1124.

  5. Tamborini G, Pepi M, Galli CA, et al. Feasibility and accuracy of a routine echocardiographic assessment of right ventricular function. Int J Cardiol. 2007;115(1):86-89.

  6. Jurcut R, Giusca S, La Gerche A, Vasile S, Ginghina C, Voigt J. The echocardiographic assessment of the right ventricle: What to do in 2010? European Journal of Echocardiography. 2010;11(2):81-96.

  7. Ho SY, Nihoyannopoulos P. Anatomy, echocardiography, and normal right ventricular dimensions. Heart. 2006;92 Suppl 1:i2-13.

  8. Abe S, Yoshihisa A, Takiguchi M, et al. Liver dysfunction assessed by model for end-stage liver disease excluding INR (MELD-XI) scoring system predicts adverse prognosis in heart failure. PloS one. 2014;9(6):e100618.

  9. van Deursen VM, Damman K, Hillege HL, van Beek AP, van Veldhuisen DJ, Voors AA. Abnormal liver function in relation to hemodynamic profile in heart failure patients. J Card Fail. 2010;16(1):84-90.

  10. Goldstein JA, Harada A, Yagi Y, Barzilai B, Cox JL. Hemodynamic importance of systolic ventricular interaction, augmented right atrial contractility and atrioventricular synchorny in acute right ventricular dysfunction. J Am Coll Cardiol. 1990;16(1):181-189.

  11. Eichhorn EJ, Konstam MA, Weiland DS, et al. Differential effects of milrinone and dobutamine on right ventricular preload, afterload and systolic performance in congestive heart failure secondary to ischemic or idiopathic dilated cardiomyopathy. Am J Cardiol. 1987;60(16):1329-1333.

  12. Feneck RO, Sherry KM, Withington PS, Oduro-Dominah A, European Milrinone Multicenter Trial Group. Comparison of the hemodynamic effects of milrinone with dobutamine in patients after cardiac surgery. J Cardiothorac Vasc Anesth. 2001;15(3):306-315.

  13. Rich S, Seidlitz M, Dodin E, et al. The short-term effects of digoxin in patients with right ventricular dysfunction from pulmonary hypertension. Chest. 1998;114(3):787-792.

  14. Harjola V, Mebazaa A, Čelutkienė J, et al. Contemporary management of acute right ventricular failure: A statement from the heart failure association and the working group on pulmonary circulation and right ventricular function of the european society of cardiology. European journal of heart failure. 2016;18(3):226-241.

  15. Ghignone M, Girling L, Prewitt RM. Volume expansion versus norepinephrine in treatment of a low cardiac output complicating an acute increase in right ventricular afterload in dogs. Anesthesiology. 1984;60(2):132-135.

  16. Tayama E, Ueda T, Shojima T, et al. Arginine vasopressin is an ideal drug after cardiac surgery for the management of low systemic vascular resistant hypotension concomitant with pulmonary hypertension. Interactive cardiovascular and thoracic surgery. 2007;6(6):715-719.

  17. Adhikari NK, Dellinger RP, Lundin S, et al. Inhaled nitric oxide does not reduce mortality in patients with acute respiratory distress syndrome regardless of severity: Systematic review and meta-analysis. Crit Care Med. 2014;42(2):404-412.

  18. Wilcox SR, Kabrhel C, Channick RN. Pulmonary hypertension and right ventricular failure in emergency medicine. Ann Emerg Med. 2015;66(6):619-628.

NO_NAME-30.png
 

Colin McCloskey, MD

University of Michigan, Critical Care Fellow

 


How to Cite this Post

[Peer-Reviewed, Web Publication]   Ray K, Parmar M (2018, August 13). Pulmonary hypertension in the ED.  [NUEM Blog. Expert Commentary by McCloskey C]. Retrieved from http://www.nuemblog.com/blog/PH


Posts You May Also Enjoy


Resources

  1. Tintinalli, Judith E., et al. “Pulmonary Hypertension.” Tintinalli's Emergency Medicine: a Comprehensive Study Guide, McGraw-Hill Education, 2016, pp. 409–412.

  2. Wilcox, Susan, et al. “Pulmonary Hypertension and Right Ventricular Failure in Emergency Medicine.” Annals of Emergency Medicine, Mosby, 3 Sept. 2015, www.sciencedirect.com/science/article/pii/S0196064415011154.

  3. Bright, Justin. “The Crashing Pulmonary Hypertension Patient.” EmDOCs.net - Emergency Medicine Education, 16 Oct. 2015, www.emdocs.net/the-crashing-pulmonary-hypertension-patient/.

  4. https://phassociation.org/wp-content/uploads/2017/02/school-resource-guide-Emergency-101-for-EMTs.pdf

Posted on August 13, 2018 and filed under Pulmonary.

Must Not Miss Fractures in the ED

Screen Shot 2018-08-05 at 8.07.44 PM.png

Written by:  MTerese Whipple , MD (NUEM PGY-3) Edited by: Ashley Amick, MD (NUEM alum '18) Expert commentary by: Matthew Pirrotte, MD


Undiagnosed fractures occur frequently in the Emergency Department setting, with a total miss rate of 1-3%.  These missed fractures not only lead to poor patient outcomes, but also account for the second highest cost of litigation against EM docs, behind only MI.1,2  This may not seem relevant if you are lucky enough to have access to a Radiologist 24-7,  however there are several injuries that will be missed if they are not included in the differential diagnosis, because even the best radiologist can’t read a film if it wasn’t ordered. This blog post will cover three ‘must not miss’ injuries to keep in mind when assessing your run-of-the-mill orthopedic injuries namely:  the Maissoneuve fracture, Lisfranc injury, and Galeazzi/Montaggia fracture-dislocations.  Finding these tricky injuries require additional radiographic views beyond those standardly ordered, but keeping them in your differential will mean better outcomes for you and your patients.


Massonieuve Fracture:

What is it and how will it present?

 A Massonieuve Fracture (which can be as difficult to pronounce as it is to miss) is a spiral fracture of the proximal 1/3 of the fibula with a disruption of the distal tibiofibular syndesmosis, which occurs in 5% of ankle injuries3. The injury occurs with pronation and external rotational forces are applied to a fixed foot, with damage propagating from the stressed tibial bone or deltoid ligament up through the interosseus membrane, causing a fracture to the proximal fibula.4 A twisted ankle in high heels is a classic mechanism for his injury.  In some cases the only apparent deformity is soft tissue swelling, pain, or ecchymosis at the ankle.  Patients may complain only of ankle pain, and because they are unable to bear weight they don’t load the damaged fibula, and therefore do not complain of lateral leg pain.  

Exam:

The patient will likely have pain with palpation over the ankle fracture/injured ligaments. Evaluate the ankle syndesmosis with compression and dorsiflexion eversion testing (will simulating a “high ankle” syndesmotic injury). In addition, make sure to palpate the proximal fibula both directly along the proximal shaft and head, and with gentle squeezing of the proximal leg just below the knee joint (a squeeze test).  Pain with these maneuvers should prompt additional radiographs.  Finally, test peroneal nerve function with ankle dorsiflexion and dorsal foot sensation. It is subject to injury in fibular fracture.

 

Radiologic Findings:

View you may not think of: Tib-fib or knee XR

Ankle AP:

Look for fractures of the medial malleolus or posterior margin of the tibia. Also look for avulsion fractures indicating interosseus ligament disruption, such as in this case, with both a fracture of the lateral malleolus and a chip fracture indicated by the white arrow [3,5]. There is obvious widening of the syndesmosis.

 

 

 

Look for joint space widening (white arrow) or widening of the syndesmosis (black arrow) [6]. If patient can’t stand, you may have to perform manual stress of the joint while the radiographs are taken (as indicated in this AP).

 

 

Knee or Tib/fib:

 

Proximal fibular fracture {3}

 

 

 

 

 

 

Management and why it matters:

This fracture is considered by many to be among the most unstable ankle injuries [4].  If there is an intact mortise with no joint space widening, the patient can be casted and follow up with orthopedics. If there is joint-space widening at the ankle mortise, surgical intervention is likely required. If undiagnosed, a patient with a Massonieuve fracture may incur a host of bad outcomes including delayed orthopedic intervention, chronic pain, arthritis, and impaired mobility.


 

Lisfranc Fracture-Dislocation

What is it and how will it present?

Lisfranc injury broadly refers to disruption of the metatarsals from the tarsus, with emphasis on the second tarsometa-tarsal joint and Lisfranc ligament [7].  The Lisfranc ligament runs obliquely from the medial cuneiform to the base of the second metatarsal (see below image for a refresher on normal foot anatomy). Injuries run the spectrum from sprain to an unstable fracture/dislocation. A dislocation of the tarsometatarsal (Lisfranc) joint is often associated with fractures, most commonly at the base of the second metatarsal or cuboid bone. It is estimated that 20-40% of Lisfranc injuries are missed on initial presentation. It can be caused by diverse mechanisms of injury including direct, high-energy trauma, such as MVCs (45% of injuries), or indirect mechanisms including [8]:

  1. Forced flexion of the forefoot with a fixed hind foot (a horseback rider falling with a foot caught in a stirrup)

  2. Forced supination/pronation on a plantar flexed foot (a soccer player having their forefoot stepped on and subsequently falling)

  3. Axial load on a flexed foot (a drunken cubs fan celebrating the World Series win by jumping from Harry Caray’s statue onto a plantar flexed foot)

Physical Exam:

Pain localizes to the midfoot.  The exam may be subtle, or there may be significant swelling and deformity present. The patient can be ambulatory or unable to bear weight.  Test the joint by stabilizing the hindfoot, any twisting of the forefoot may cause pain. Compression across the forefoot will stress the space between the first and second metatarsals, causing a pain or a palpable click if a Lisfranc injury is present.  The Piano-key test is preformed by stabilizing the hindfood, grasping the metatarsals, and preforming passive dorsiflexion and plantar flexion at the tarsometatarsal joint, looking for pain or subluxation.9  Rarely they can have associated dorsalis Pedis injury as it courses near the joint, so make sure to check pulses. The tibialis anterior nerve can also become interposed and cause the big toe to point upwards, called the “Toe Up Sign.”

Radiologic Findings:

If a Lisfranc injury is suspected, foot radiographs with additional views including WEIGHT BEARING AP, lateral, and oblique are essential.

First a normal foot:

  1. The lateral margin of the 1st metatarsal should be aligned with the lateral margin of the medial cuneiform.

  2. The medial aspect of the base of the 2nd metatarsal should align with the medial border of the middle cuneiform.

  3. The medial margins of the 4th metatarsal and cuboid should be aligned [10].

 

 

 

Findings suggesting injury:

AP: Diastasis of >2 mm between the base of the 1st and 2nd metatarsals indicates Lisfranc injury. 90% have associated avulsion fracture of the base of the second metatarsal or medial cuneiform, known as Fleck Sign (pictured at left). The pictured radiograph also demonstrates lateral displacement of all 5 metatarsals [11,12].

Lateral: Allows for identification of any dorsal or plantar dislocation [12]. 

Oblique: Allows for evaluation of the alignment of the 3rd and 4th metatarsals with the cuboid and cuneiform [12]. 

 

 

 

Management and why it matters:

If there is no evidence of widening of the Lisfranc joint space, the patient can be splinted and follow up with orthopedics, however they MUST BE non-weightbearing. Any evidence of fracture-dislocation >2 mm requires orthopedic consultation in the ED for likely operative fixation. Fractures found later have worse outcomes. Delayed ORIF after late recognition is better than no intervention, however most patients still require shoe modification or orthoses [12]. 


Galeazzi and Monteggia Fracture Dislocations

The radius and ulna are joined by an interosseus membrane. When one is injured the other is likely to be affected as well (just like the tibia/fibula).

Management and why it matters: 

If either fracture is suspected, consult hand surgery/orthopedics for reduction and definitive management. Both almost always require ORIF or other surgical treatment. Chronic pain and limitation of supination and pronation can occur if not properly treated [13]. 


Expert Commentary

Drs. Whipple and Amick do a nice job of highlighting several eponymous fractures which can be tricky to diagnose. In general I find that missed extra-axial orthopedic injuries in the emergency department are the result of several factors

  1. Failure to “film what hurts.” If a patient feels that their injury was sufficiently serious to warrant a visit to the emergency department, the prudent practitioner maintains a low threshold for imaging. Clinical decision rules for judicious imaging are clearly valid but need to be applied judiciously. When in doubt, get the film.

  2. Failure to review films directly. Radiologists, while skilled and vital partners, rarely have the detailed information gleaned from simply pressing on patient’s bones and figuring out where they hurt. Correlation with point tenderness is a critical part of radiographic assessment. Scrutiny of radiographic bony anatomy near the sites of tenderness can lead to discovery of subtle fractures.

  3. Failure to consider mechanism. Given the frequency with which we in the ED see serious trauma, it is easy to fall into a trap of being unimpressed with mechanisms that are actually quite severe. Every experienced acute care practitioner has had the chance to be absolutely flabbergasted by the severe polytrauma that can result from “low impact’ mechanisms such as stair falls, falls from standing, and pedestrians struck by vehicles at low speed.

The ramifications of a missed fracture can be significant. A recent analysis of closed legal claims in emergency medicine found that three of the top ten diagnoses in medical malpractice lawsuits were related to fracture care(vertebral, radius/ulna, tibia/fibula) [14]. A similar analysis of pediatric cases demonstrated that in children over the age of 3, fractures remain the most common source of medical malpractice claims [15]. This is to say nothing of the obvious morbidity and potential disability that may result from a missed injury.

The interesting thing about the fractures that discussed by Drs. Whipple and Amick is that, at least in the case of the Maisonneuve and forearm fractures, what tends to be missed is the severity and operative nature of these injuries rather than the fractures themselves.A clinician seeing a patient with an eponymous forearm fracture will likely not misdiagnose them as an elbow sprain. Similarly, few people would interpret the ankle films of a patient with Maisonneuve fracture to be normal, the problem comes in missing the fibular injury. Lisfranc’s fracture is a different entity; it is not uncommon for these patients to be misdiagnosed several times as having a “foot sprain” before the proper diagnosis is made.

 

One thing you can take to the bank in emergency orthopedics is that if the fracture is named after someone the injury involved can usually find a way to trick even a savvy clinician. Bennett, Rolando, Jefferson, Smith, and Sagond are also names that will you will encounter in your career.  As yet no one has attached their name to the nondisplaced fracture of the distal phalanx of the small toe, but one never knows.

 

Laura_Whiteside-13.jpg

 

Matthew Pirrotte, MD

Assistant Professor of Emergency Medicine, NUEM

 

 


How to cite this post

[Peer-Reviewed, Web Publication]   Whipple M, Amick A (2018, August 6). Can't Miss Fractures in the ED.  [NUEM Blog. Expert Commentary by Pirotte M]. Retrieved from http://www.nuemblog.com/blog/missed-fractures


Posts you may also enjoy


References:

  1. Schwartz, D. Ten Most Commonly Missed Radiographic Findings in the ED. Boston Scientific Assembly. Thursday, October 8, 2009. Boston Convention & Exhibition Center.

  2. Hallas P and T Ellingsen. Errors in fracture diagnoses in the emergency department – characteristics of patients and diurnal variation. BMC Emergency Medicine. 2006. 6(4). doi:10.1186/1471-227X-6-4.

  3. Millen JC and D Lindberg. Maissoneuve Fracture. The Journal of Emergency Medicine. 2011. 41(1): 77–78.

  4. Charopoulos I, Kokoroghiannis C, Karagiannis S, Lyritis GP, Papaioannou N. Maisonneuve fracture without deltoid ligament disruption: a rare pattern of injury. The Journal of foot and ankle surgery : official publication of the American College of Foot and Ankle Surgeons. 49(1):86.e11-7

  5. Sports Medicine for the Emergency Physician: A Practical Handbook. Ed. A. Waterbrook. Cambridge University Press: NY, NY. 2016. 75-77, 130-131, 248-249, 273.

  6. Taweel NR et al. The proximal fibula should be examined in all patients with ankle injury: A case series of missed Maisonneuve fractures. The Journal of Emergency Medicine. 2013. 44(2): 251-255.

  7. Wynter S, Grigg C. Lisfranc injuries. Aust Fam Physician. 2017 Mar;46(3):116-119.

  8. Desmond EA, Chou LB. Current concepts review: Lisfranc injuries. Foot Ankle Int 2006;27(8):653–60.

  9. Seybold JD, Coetzee JC. Lisfranc injuries: When to observe, fix, or fuse. Clin Sports Med 2015;34(4):705–23.

  10. Sherief TI, Mucci B, Greiss M. Lisfranc injury: How frequently does it get missed? And how can we improve? Injury, Int. J. Care Injured. 2007. 38: 856—860.

  11. Gupta, RT et al. Lisfranc injury: Imaging findings for this important, but often missed diagnosis. Curr Probl Diagn Radiol. 2008 May/June. 115-126.

  12. van Rijn J et al. Missing the Lisfranc Fracture: A case report and review of the literature. The Journal of Foot & Ankle Surgery. 2012. 51: 270-274.

  13. Perron, A et al. Orthopedic pitfalls in the ED: Galeazzi and Monteggia Fracture-Dislocation. Am J Em Med. 2001 May. 19(3): 225-228.

  14. Brown, T. W., McCarthy, M. L., Kelen, G. D. and Levy, F. (2010), An Epidemiologic Study of Closed Emergency Department Malpractice Claims in a National Database of Physician Malpractice Insurers. Academic Emergency Medicine, 17: 553–560

  15. Selbst SM, Friedman MJ, Singh SB. Epidemiology and etiology of malpractice lawsuits involving children in US emergency departments and urgent care centers. Pediatr Emerg Care. 2005 Mar; 21 (3): 165-169


Posted on August 6, 2018 and filed under Orthopedics.

Delirium as a symptom of UTI: physiology or pseudoaxiom?

Screen Shot 2018-07-08 at 3.00.34 PM.png

Written by: Ashley Amick, MD (NUEM alum '18) Edited by: Michael Macias, MD (NUEM alum '17) Expert commentary by: Alexander S Lo, MD, PhD


Asymptomatic bacteriuria (ASB) is a prevalent condition in the elderly population.  Bacterial colonization of the genitourinary (GU) tract increases with age and institutionalized status.  Though once thought to be pathogenic, randomized trials clearly demonstrate that treatment of ASB with antibiotics does not improve outcomes, except in pregnant patients and those undergoing GU procedures.  Emerging data even suggest there may be a protective effect of colonizing bacteria.  Conversely, there is increasing recognition of the dangers of inappropriate antibiotic use, both to the individual and the general population, and widespread agenda to limit unnecessary antimicrobial use. 

As the antibiotic stewardship movement marches forward, the treatment of ASB continues to be a central focus.  Recent guidelines emphasize that the presence of lower GU symptoms is the key distinction between UTI and ASB.  This strategy may be easily adopted in young otherwise healthy patients, but reaches a major stumbling block when considering the elderly population.  This is in part due to the fact that many clinicians believe that there is a causative relationship between UTI and delirium in the absence of other localizing symptoms or signs of systemic infection.  In other words, delirium is the symptom that substantiates a diagnosis of UTI in the presence of otherwise asymptomatic bacteriuria.  This concept, now generations old, is still taught in many medical school curricula.  The correlation between delirium and UTI is so well established in the minds of clinicians that many have never questioned whether this presumed association is rooted in data.

The concerning truth is that there is no reliable evidence to suggest that such a relationship between delirium and UTI exist.  A recent review of the literature found only five papers addressing this association primarily, all were observational and therefore lacked the ability to make conclusions about the degree of causation.  All studies were severely methodologically flawed, and none were case-control, cohort, or RTCs.  Additionally, there is no physiologic evidence or models to suggest that bacteriuria in the absence of systemic illness, results in cognitive dysfunction.  No known studies have ever shown that treatment of otherwise asymptomatic bacteriuria improves delirium outcomes.  Taking these data into account, the CDC and SHEA created guidelines specifically do not include delirium as a reason to treat potential UTIs in non-catheterized patients.  These represent a departure from earlier guidelines that included altered mental status as a symptom of UTI in the elderly.  The new SHEA recommendations have been tested in a large randomized trail and were found to be safe when compared to standard care.

Despite efforts to shift practice patterns in the direction of a more guideline-based management, ASB continues to be unnecessarily treated at high rates in the elderly.  One reason may be that anecdote is a powerful source of bias.  Many clinicians support their belief of a causative correlation between UTI and delirium by referencing cases where patient presented with confusion and were found to have a UTI.  The problem is, how was that “UTI” diagnosed?  The distinction is more than just semantics.  In the absence of GU symptoms and signs of systemic infection, then the clinician made the diagnosis solely on the basis of a UA and urine culture.  But as previously discussed, both a UA and culture will frequently be positive in both ABS and UTI, and cannot reliably distinguish between the two conditions. 

Many clinicians will cite the fact that the patients may improve following antibiotic administration, thereby confirming their suspicion of a presumed UTI-related delirium.  However, delirium frequently is short lived and self-resolving, therefore improvement is likely to be simply coincidental.  In addition, along with antibiotics administration patients also often receive intravascular volume, thereby improving hydration status, which is a frequent cause of delirium.  These factors confound the ability of the clinician to objectively interpret the causative relationship between the delirium and bacteriuria.  High quality randomized trials will be needed to further clarify these issues and assess is the high rate of concurrence of bacteriuria and delirium is due to causation or simply coincidence.


Expert Commentary

Over 50 million U.S. adults > 65 years of age (“older adults”), account for over 20 million Emergency Departments (ED) visits each year [1].  Many of these patients have unmet and complex underlying medical needs that are often understated by their chief complaints. The tempting application of traditional ‘one complaint; one algorithm’ approach taught to many emergency physicians, may often result in long-term, downstream, adverse outcomes.  One of those relevant to the accompanying blog, is the traditional “if grandma is delirious, look for and treat the UTI” doctrine.  A review of the literature proves that the evidence linking UTI’s to delirium in older adults is lacking [2]. Many older adults are bacteriuric; most do NOT have to be treated [3].  The delirium is not a reason to treat bacteriuria [4].  It is also just as likely that it is the other comorbid conditions causing the delirium, since 75% of older adults have two or more comorbid chronic conditions [5]. many of which have the potential to cause delirium at any time[6].   The patient may likely require admission for the delirium, but a more comprehensive investigation into its etiology is more helpful than treating the easy target of a contaminated urine sample

Alex_Lo.png

Alexander S Lo, MD, PhD

Assistant Professor of Emergency Medicine, Northwestern University 


Posts you may also enjoy


How to cite this post

[Peer-Reviewed, Web Publication]   Amick A, Macias M (2018, July 30). Delirium as a symptom of UTI: physiology or pseudoaxiom.  [NUEM Blog. Expert Commentary by Lo A]. Retrieved from http://www.nuemblog.com/blog/uti-part1


Resources

  1. Pines JM, Mullins PM, Cooper JK, Feng LB, Roth KE. National trends in emergency department use, care patterns, and quality of care of older adults in the United States. Journal of the American Geriatrics Society. 2013;61(1):12-17.

  2. Balogun SA, Philbrick JT. Delirium, a Symptom of UTI in the Elderly: Fact or Fable? A Systematic Review. Canadian geriatrics journal : CGJ. 2014;17(1):22-26.

  3. Finucane TE. "Urinary Tract Infection"-Requiem for a Heavyweight. Journal of the American Geriatrics Society. 2017;65(8):1650-1655.

  4. Ninan S. Don't assume urinary tract infection is the cause of delirium in older adults. Bmj. 2013;346:f3005.

  5. Working Group on Health Outcomes for Older Persons with Multiple Chronic C. Universal health outcome measures for older persons with multiple chronic conditions. Journal of the American Geriatrics Society. 2012;60(12):2333-2341.

  6. Kuluski K, Hoang SN, Schaink AK, et al. The care delivery experience of hospitalized patients with complex chronic disease. Health expectations : an international journal of public participation in health care and health policy. 2013;16(4):e111-123.

  7. McKenzie, Robin, et al. "Bacteriuria in individuals who become delirious." The American journal of medicine 127.4 (2014): 255-257.

  8. Balogun, Seki A., and John T. Philbrick. "Delirium, a symptom of UTI in the elderly: fact or fable? a systematic review." Canadian Geriatrics Journal 17.1 (2013): 22-26.

  9. Nace, David A., Paul J. Drinka, and Christopher J. Crnich. "Clinical uncertainties in the approach to long term care residents with possible urinary tract infection." Journal of the American Medical Directors Association 15.2 (2014): 133-139.

  10. Gau, Jen-Tzer, et al. "Interexpert agreement on diagnosis of bacteriuria and urinary tract infection in hospitalized older adults." J Am Osteopath Assoc 109.4 (2009): 220-226.

  11. Juthani-Mehta, Manisha, et al. "Interobserver variability in the assessment of clinical criteria for suspected urinary tract infection in nursing home residents." Infection Control & Hospital Epidemiology 29.05 (2008): 446-449.

  12. Schulz, Lucas, et al. "Top Ten Myths Regarding the Diagnosis and Treatment of Urinary Tract Infections." The Journal of emergency medicine (2016).

 

Posted on July 30, 2018 and filed under Infectious Disease.

A Deep "Seeded" Cough

Screen Shot 2018-07-08 at 2.48.04 PM.png

Written by: Laurie Nosbusch, MD (NUEM PGY-2) Edited by: Jon Andereck, MD, (NUEM PGY-4) Expert commentary by: Viren Kaul, MD


The Case

Chief Complaint: Cough for 3 weeks

History of Present Illness: An 18 month old male patient presents to the emergency department for a cough that has persisted for 3 weeks. He had a runny nose for one week, but the cough has persisted for two weeks beyond resolution of his congestion. The cough is non-productive and is worse at night when lying down. Parents deny fever, shortness of breath, wheezing, vomiting, choking, change in energy level, change in appetite, or weight loss.

Physical Exam:

Vitals: T 37.2, HR 103, BP 92/palp, RR 35, Sat 97% on room air

General: Well-appearing, interactive, sitting with parents

Pulmonary: Tachypneic but no signs of respiratory distress, no stridor, no accessory muscle use, no retractions, no tracheal tugging, no nasal flaring. Right lung is clear to auscultation. Left lung has decreased breath sounds at the base.

The rest of the physical exam is unremarkable.

 

Chest X- Ray [1]:

Figure 1. Radiology interpretation: Hyperlucency of left lung with mediastinal shift to right

Figure 1. Radiology interpretation: Hyperlucency of left lung with mediastinal shift to right

Differential Diagnosis: Bronchial mass, congenital lobar emphysema, foreign body aspiration

Case Resolution: Bronchoscopy was performed and food debris (possibly a seed or popcorn) was removed from the left lower bronchus. The left mainstem bronchus was inflamed. The patient was treated for post-obstruction inflammation and pneumonia with steroids and antibiotics.

Final Diagnosis: Foreign body aspiration


Discussion:

Foreign Body Aspiration Causing Partial Airway Obstruction  

Epidemiology: Foreign body aspiration is a common presentation in the emergency department. Nearly 80% of these events occur in children younger than 3 years and they are more common in males. [2]

Presentation: These patients may have variable presentations depending on timing:

Screen Shot 2018-07-20 at 10.58.51 AM.png

Common Chest X-Ray Findings:

  • Visualization of radio-opaque foreign body

  • Normal chest x-ray (30%) [7,8]

  • Lower airway obstruction: hyperinflated lung, hyperlucent lung, atelectasis, mediastinal shift, pneumonia, abscess [7,9] (See Figure 1 Above)

  • Lower airway obstruction: hyperinflated lung, hyperlucent lung, atelectasis, mediastinal shift, pneumonia, abscess [7,9] (See Figure 1 Above)

  • Lateral decubitus films: air trapping due to foreign body in bronchus prevents collapse of affected lung [9] (Compare Figures 2 and 3 Below)

Figure 2. Left Lateral Decubitus Film [9]Left lung collapses when in dependent position. This is normal and does not suggest foreign body or air trapping in left lung.

Figure 2. Left Lateral Decubitus Film [9]

Left lung collapses when in dependent position. This is normal and does not suggest foreign body or air trapping in left lung.

Figure 3. Right Lateral Decubitus Film [9]Right lung does not collapse when in dependent position. This is abnormal and suggests foreign body in right bronchus causing air trapping in right lung.

Figure 3. Right Lateral Decubitus Film [9]

Right lung does not collapse when in dependent position. This is abnormal and suggests foreign body in right bronchus causing air trapping in right lung.

Management:

  • Address the ABCs

  • Obtain a history, specifically asking about choking events

  • If the history is concerning for foreign body aspiration or if breath sounds are asymmetric, order x-ray

    • PA/lateral chest views

    • Consider expiratory phase chest x-ray (in cooperative patients) or bilateral decubitus chest x-rays (for younger, less cooperative patients) as these can enhance detection of unilateral air trapping [9]

    • If there is concern for laryngotracheal foreign body, obtain neck PA/lateral x-rays

    • If the x-rays are negative, order CT or proceed directly to bronchoscopy depending on clinical suspicion [9]

  • Bronchoscopy is performed to remove the foreign body

  • If there is evidence of inflammation or infection, give steroids and/or antibiotics

  • Initial empiric antibiotics should cover oral anaerobes, ex. ampicillin-sulbactam [10]


Key Points:

  • Consider foreign body aspiration for any pediatric patient with a respiratory complaint

  • The H&P is important for diagnosis because chest x-rays can be normal 30% of the time

  • Think about foreign body aspiration when you see an x-ray suggestive of air trapping


Expert Commentary

Thank you for the opportunity to review this well summarized article on partial tracheobronchial foreign body aspiration (FBA) in pediatric patients.

Here are my TOP TEN TIPS:

  1. Beware the unwitnessed FBA!

  2. Ask for presence of an older sibling, they often provide the foreign body (FB) to the younger sibling.

  3. > 60% FBs land on the right side and > 80% are organic.

  4. Having an iron will is a good thing. Having an iron pill in your airway: bad! Iron and potassium tablets dissolve in the airway, cause severe inflammation and result in stenosis.

  5. Smaller the child, smaller the airway, more likelihood of obstruction.

  6. Which FBs are most likely to cause obstruction and be fatal? MNEMONIC: They reach the RIBS!!

    1. Round

    2. Incompressible

    3. Don’t Break easily

    4. Smooth

  7. Peanuts are commonest single food item responsible for FBA.

  8. DO NOT conduct blind sweeps of the mouth as it can lead to complete airway obstruction.

  9. What to do should the child stop speaking or coughing i.e. develop a complete central airway obstruction?

    1. Ask for help!

    2. Infants: Alternating back blows and compressions

    3. Older children/adults: Heimlich maneuver

    4. Follow the AHA guidelines

  10. Bronchoscopy is recommended in all cases where the suspicion for FBA is high. In children, rigid bronchoscopy is recommended. Flexible bronchoscopy can be used for diagnosis in uncertain situations but having a rigid bronchoscope in standby is strongly advised.

Picture1.png

Viren Kaul, MD

Fellow, Pulmonary and Critical Care Medicine

Mount Sinai School of Medicine at Elmhurst


Posts you may also enjoy


How to cite this post

[Peer-Reviewed, Web Publication]   Nosbusch L, Andereck J (2018, July 23). A deep "seeded" cough.  [NUEM Blog. Expert Commentary by Kaul V]. Retrieved from http://www.nuemblog.com/blog/pediatric-cough


References:

  1. Case courtesy of Dr Jeremy Jones, <a href="https://radiopaedia.org/">Radiopaedia.org</a>. From the case <a href="https://radiopaedia.org/cases/26866">rID: 26866</a>

  2. Tan HK, Brown K, McGill T, et al. Airway foreign bodies (FB): a 10-year review. Int J Pediatr Otorhinolaryngol 2000; 56:91.

  3. Wiseman NE. The diagnosis of foreign body aspiration in childhood. J Pediatr Surg 1984; 19:531.

  4. Laks Y, Barzilay Z. Foreign body aspiration in childhood. Pediatr Emerg Care 1988; 4:102.

  5. Blazer S, Naveh Y, Friedman A. Foreign body in the airway. A review of 200 cases. Am J Dis Child 1980; 134:68.

  6. Mu L, He P, Sun D. The causes and complications of late diagnosis of foreign body aspiration in children. Report of 210 cases. Arch Otolaryngol Head Neck Surg 1991; 117:876.

  7. Sahin A, Meteroglu F, Eren S, Celik Y. Inhalation of foreign bodies in children: experience of 22 years. J Trauma Acute Care Surg 2013; 74:658.

  8. Svedstrom E, Puhakka H, Kero P. How accurate is chest radiography in the diagnosis of tracheobronchial foreign bodies in children? Pediatr Radiol 1989;19:520.

  9. Laya BF, Restrepo R, Lee EY. Practical imaging evaluation of foreign bodies in children: an update. Radiol Clin N Am 2017; 55:845

  10. Sandora TJ, Harper MB. Pneumonia in hospitalized children. Pediatr Clin North Am 2005; 52:1059.

Posted on July 23, 2018 and filed under Pulmonary.

Non-Invasive Positive Pressure Ventilation in the ED

Screen Shot 2018-07-08 at 2.31.23 PM.png

Written by: Matt McCauley, MD (NUEM PGY-2) Edited by: Sarah Sanders, MD, (NUEM PGY-4) Expert commentary by: James Walter, MD


Noninvasive positive pressure ventilation (NIPPV) refers to the delivery of positive pressure ventilator support without the insertion of an endotracheal tube [1]. This intervention works to improve lung volumes and decrease the work of breathing, making it a practical tool in the management of acute respiratory failure [2]. Due to the multitude of indications, it is important for emergency physicians to understand both the ventilator settings of NIPPV devices and the types of respiratory failure they address. 

Fig 1


Approach to the Patient

The utilization of NIPPV requires active management by the EM provider. One cannot simply set the patient on initial settings of “ten over five” and walk away; both subjective criteria (eg patient comfort, patient mental status, and degree of air leak around mask) and objective data (eg O2 saturation, respiratory rate, pH, PaCo2) must be taken into account. The provider can start promoting success at the initiation of treatment by starting at low settings and talking the patient through the procedure, both of which can improve compliance [5]. If time permits, baseline blood gases obtained at this point can be useful in monitoring clinical course [10].

Fig 2

Different etiologies of respiratory failure, as described in Figure 2, require different approaches to the titration of ventilator settings. In the case of a patient with an acute exacerbation of COPD, the clinician should initially adjust FiO2 to an O2 saturation of 88-92%, taking care to avoid chasing high saturations that can paradoxically increase shunt, decrease respiratory drive, and subsequently promote patient deterioration. Arterial blood gas measurements should then be taken at thirty minutes and then trended over 1-2 hours of therapy [5]. If the patient continues to demonstrate failure to blow off CO2 or has not improved tidal volumes, ventilation can be improved by increasing IPAP alone while keeping EPAP constant, thereby improving tidal volumes, oxygenation, and CO2 retention [7,10].

Patients with pulmonary edema exhibit type 1 failure and require a different approach. The pathophysiology of pulmonary edema causes alveoli to be less available for gas exchange as the lungs are filled with fluid, leading to a shunt physiology with alveoli being perfused but not able to oxygenate or ventilate. This shunt physiology manifests itself as a low O2 saturation despite the use of 100% FiO2. This requires an increase in mean alveolar pressure to correct which is best accomplished by increasing the IPAP and EPAP in tandem which forces fluid out of the alveoli by an increase in the overall mean alveolar pressure [9,10].  This increase in pressures must done slowly to balance the need for increased pressures against patient comfort and the limit of recruitable alveoli. Persistent need for EPAP pressures 10-12cm H20 should push management toward intubation [10].


Expert Commentary

Thank you for the opportunity to review this helpful post. As you mention, non-invasive positive pressure ventilation (NPPV) is a potentially life-saving supportive therapy for patients with acute respiratory failure. Emergency Medicine providers should be familiar with when and how to use this important tool.

 If I were to highlight just one thing in your post, it would be your suggestion to “start monitoring.” This should be in bold and in 30-point font.

Attentive bedside monitoring of patients recently placed on NPPV matters exponentially more than any other aspect of therapy.

NPPV can decrease work of breathing, improve oxygenation, improve alveolar ventilation, and counteract auto-PEEP. All of these can and should be monitored at the bedside as the pressure requirements to achieve these goals will differ with each patient depending on the mechanics of their respiratory system and the severity of their disease. Close bedside monitoring is also essential to determine if a patient is failing a trial of NPPV and requires invasive mechanical ventilation. When returning to the room, you should be asking yourself the following: Has my patient’s work of breathing improved? Is my patient still hypoxemic? Is their respiratory acidosis better? Are they having difficulty with secretions? How is their mental status? Many studies show that delaying intubation, when ultimately necessary, worsens outcomes so it is critical to recognize a failing patient early and take control of the situation. I think it’s often helpful to set a clear time limit with NPPV, for instance “I am going to trial NPPV in this patient with acute decompensated heart failure (ADHF). If his work of breathing and RR remain high in 20 minutes, we will move towards intubation.” In general, if you place a patient on NPPV in the emergency department (ED), you should plan to return to their bedside frequently over the next 45 minutes. Make this part of your practice.

A few points on terminology since it’s confusing:

  • Expiratory positive airway pressure (EPAP) on NPPV is the same as positive end-expiratory pressure (PEEP) when using invasive mechanical ventilation.
  • Continuous positive airway pressure (CPAP): an NPPV mode where the machine delivers a continuous level of airway pressure (e.g., on CPAP 5, the machine will continuously deliver 5 cmH20 during inspiration and expiration). Breaths in this mode are all patient triggered (an apneic patient will remain apneic on CPAP) and not supported with any additional pressure support.
  • Bilevel positive airway pressure (BPAP): an NPPV mode where you set an EPAP and an inspiratory positive airway pressure (IPAP). Breaths in this mode are patient-triggered (an apneic patient placed on BPAP will remain apneic unless your machine has a backup rate), pressure-targeted (the machine delivers the set IPAP with each patient-triggered breath), and flow-cycled (the IPAP is delivered until the machine senses a set % decrease in patient inspiratory flow at which point the pressure drops back to EPAP and the patient passively exhales). As this is a pressure mode, you do not directly control the tidal volume; instead it is determined by patient effort, respiratory system mechanics, and the difference between IPAP and EPAP (also known as the driving pressure or pressure support). A higher driving pressure (a bigger difference between IPAP and EPAP) will produce a bigger tidal volume.
  • BiPAP and BIPAP: these are two proprietary modes of BPAP (the first by Respironics and the second by Drager). It’s unnecessarily confusing, I know, but just be aware that BiPAP and BIPAP are brand names, BPAP is the generic term which you should be using.
  • On BPAP, airway pressure cycles from the set EPAP to the set IPAP (e.g., on BPAP 15/5, the pressure will cycle from 5 cmH20 to 15 cmH20 with each breath). On invasive mechanical ventilation in the pressure control mode, you don’t set an IPAP but rather a desired level of pressure support (PS). This is the pressure above PEEP. So on PS 15/5, the pressure will cycle from 5 cmH20 to 20 cmH20 (15 cmH20 above PEEP). In other words, BPAP 15/5 will generate the same pressures as PS 10/5.

 Some basic suggestions on settings:

  • EPAP and IPAP settings can be adjusted in increments of 2-3 q 5 minutes as needed
  • Titrate EPAP to achieve the desired O2 saturation (aim for >88% in COPD pts who are chronic CO2 retainers).
  •  As noted, the level of PS is defined as IPAP-EPAP; increased IPAP-EPAP=increased tidal volume/increased ventilation.
  • Begin with IPAP 5 cmH2O above EPAP (to provide 5 cmH2O of PS); increase IPAP-EPAP as needed, titrated to lessen the RR, lessen the visible work of breathing, and decrease PCO2 in hypercapnic pts 
  • Remember that whenever you increase EPAP you have to increase IPAP by a similar amount to maintain the same level of PS (e.g., if inadequate oxygenation: change 10/5 to 13/8 to keep a PS of 5 cmH20).
  •  In general, EPAP should not exceed 8-10 cmH2O and IPAP not exceed 20 cmH2O (this level of support should make you strongly consider intubation).
  •  Titrate FiO2 down to ≤60% as long as adequate O2 saturation is maintained.
  •  EPAP/PEEP: In addition to decreasing preload and reducing airway collapse at end-expiration as you mention, EPAP/PEEP also counteracts the effects of auto-PEEP (which helps decrease work of breathing in severe COPD/asthma) and decreases left ventricular afterload.

Just to be clear, NPPV does not a have strong evidence base in all forms of pulmonary edema, only hydrostatic/cardiogenic pulmonary edema (ADHF). In ADHF, NPPV (especially the EPAP part) works as an LV assist device by dropping LV preload and decreasing LV afterload. Whether you place a patient in ADHF on CPAP or BPAP doesn’t seem to matter much. This was best studied in a 2008 NEJM trial that did not show any clear benefit to BPAP vs CPAP (although both were better than standard O2). It is important to remember that the use of NPPV/EPAP may cause clinical deterioration in patients with right ventricular failure. EPAP increases RV afterload and drops RV preload so close bedside monitoring is essential if using NPPV in patients with RV failure.

ARDS is also a pulmonary edema syndrome (edema in ARDS is caused by disruption of the alveolar epithelial/endothelial barrier) but the evidence for NPPV is much weaker than in ADHF. Based on some recent trials, many of us are moving towards high-flow nasal cannula in this setting rather than NPPV (reviewed in detail here).

 A small semantic point: Throughout your review, you mention monitoring “compliance.” Generally, “compliance” denotes a patient’s willingness to follow treatment recommendations. “Non-compliance” tends to be a negative term; a patient knows what they should do but chooses to do otherwise. What you are assessing when using NPPV in the ED is not “compliance” but “tolerance.” In 99% of cases, the factors that limit use of NPPV in acutely ill patients in the ED are not within a patient’s control: fear, anxiety, delirium, vomiting, feeling like they are unable to breathe or get enough air, etc.

Talk with RT and your program leadership to find a time to trial NPPV. Clinicians who use NPPV should know what a high EPAP or driving pressure feels like so you can better coach your patients through what they are going to experience when starting therapy.

James "Mac" Walter

Instructor of Medicine, Pulmonary and Critical Care


How to cite this post

[Peer-Reviewed, Web Publication]   McCauley M, Sanders S (2018, July 16 ). Non-invasive positive pressure ventilation in the emergency department.  [NUEM Blog. Expert Commentary by Walter J]. Retrieved from http://www.nuemblog.com/blog/NIPPV


Posts you may also enjoy


References

  1. Cabrini L, Landoni G, Oriani A, et al. Noninvasive ventilation and survival in acute care settings: A comprehensive systematic review and metaanalysis of randomized controlled trials. Crit Care Med 20 2015 Apr;43(4):880-8
  2. Carlson JN, Wang HE. Noninvasive Airway Management. In: Tintinalli JE, Stapczynski J, Ma O, Yealy DM, Meckler GD, Cline DM. eds. Tintinalli’s Emergency Medicine: A Comprehensive Study Guide, 8e New York, NY: McGraw-Hill; 2016
  3. Confalonieri  M, Potena  A, Carbone  G, Porta  RD, Tolley  EA, Umberto Meduri G. Acute respiratory failure in patients with severe community acquired pneumonia. A prospective randomized evaluation of noninvasive ventilation. Am J Respir Crit Care Med. 1999;160(5 Pt 1):1585–1591
  4.  Keenan SP Mehta S. Noninvasive ventilation for patients presenting with acute respiratory failure: the randomized controlled trials. Respir Care 2009;54:116–26
  5.  Kelly CR, Higgins AR, Chandra S. Noninvasive positive-pressure ventilation. N Engl J Med 2015;372:e30-e30
  6.  Liesching T, Kwok H, Hill NS. Acute applications of noninvasive positive pressure ventilation. Chest 2003; 124: 699–713.
  7. LIGHTOWLER JVJ, ELLIOTT MWPredicting the outcome from NIV for acute exacerbations of COPD Thorax 2000;55:815-816
  8. Lim WJ, Mohammed Akram R, Carson KV, Mysore S, Labiszewski NA, Wedzicha JA, Rowe BH, Smith BJ. Non-invasive positive pressure ventilation for treatment of respiratory failure due to severe acute exacerbations of asthma. Cochrane Database of Systematic Reviews 2012, Issue 12.
  9.  Vital FM, Ladeira MT, Atallah AN. Non-invasive positive pressure ventilation (CPAP or bilevel NPPV) for cardiogenic pulmonary edema. Cochrane Database Systematic Reviews  2013 Issue 5
  10. Wright BJ, Slesinger TL. Noninvasive Positive Pressure Ventilation. In: Farcy DA, Chiu WC, Marshall JP, Osborn TM. eds. Critical Care Emergency Medicine, 2e New York, NY: McGraw-Hill
Posted on July 16, 2018 and filed under Pulmonary.

Approach to Hypothermic Resuscitation

Screen Shot 2018-05-31 at 11.50.30 AM.png

Written by:  Luke Neil, MD (NUEM PGY-2) Edited by: Quentin Rueter, MD, (NUEM PGY-4) Expert commentary by: Kory Gebhardt, MD


Hypothermia-page-001.jpg

Expert Commentary

This is a good overview of the algorithmic approach to the hypothermic patient. Generally speaking, hypothermia can be divided into various categories of severity, but as you mention, it is really those patients with a core temperature of <32°C (90°F) with cardiac instability or cardiac arrest that will require especially aggressive care.

For any hypothermic patient, the most important initial intervention is to stop any further heat loss. This is especially important for those with damp or wet clothing. Any wet garments should be completely removed, the patient should be dried, and then covered with warm, dry blankets and possibly a forced air rewarming device (i.e. Bair Hugger). Recall that one of the most efficient ways to cool a HYPERthermic patient is with evaporative cooling (spraying with or submerging them in water and then using fans to circulate air over the wet surfaces). Similarly, this heat loss will strongly work against you in rewarming a hypothermic patient if they are not fully dry. After this simple intervention, the majority of mildly hypothermic and stable patients just need time to bring their core temperature back to normal and often can be discharged once this has occurred.

For those patients with a core temp >32°C with severe cardiac instability or in cardiac arrest, you should also consider alternative etiologies for their presentation rather than expect it solely caused by the hypothermia alone. Like you mention, if you are able to rewarm a cardiac arrest patient above this temperature and they remain in asystole, it is likely that irreversible damage has occurred and they are less likely to be able to be successfully resuscitated.

As you detail in the algorithm, those with a temperature less than 32°C (90°F) AND instability or arrest need aggressive and invasive rewarming. The best available means of doing this is ECMO. Much of the research surrounding accidental hypothermia and resuscitation comes from the Nordic countries where freezing temperatures are often combined with outdoor extracurriculars and results in a high “n” for the studies. Outcomes data from many of the expert centers in this area show major benefits of ECMO, including one showing survival post-arrest in nearly 60% of patients and, even more importantly, good neurologic outcomes in 38% compared to only 3% in those without extracorporeal rewarming!

Unfortunately, not all EM physicians will have quick or 24/7 availability of ECMO. While this should be the preferred means of rewarming if available, there are alternatives if it is not. Hemodialysis circuits can also be used to actively rewarm a patient. Generally these can achieve 2-4 degrees/hr of rewarming compared to the 4-6 degrees/hr of ECMO. Thoracic (bilateral chest tubes), gastric (NG tube), and bladder lavage (foley) with warm fluids can also provide several degrees per hour of rewarming if used appropriately. Use a ventilator that can warm and humidify air. Don’t forget about minimizing heat loss by fully drying the patient and keeping as much of them covered as possible.

Lastly, I want to say a word about prognostication. While the mantra is, “you’re not dead until you’re warm and dead”, you can imagine that these patients require a considerable amount of time, effort, and mobilization of resources when they present to the ED. There is information that can help guide which patients are likely to benefit from such aggressive care from those who are, unfortunately, unlikely to be resuscitated. While multiple markers have been studied, the one with the most evidence supporting it, is a potassium value. This value can serve as a sort of surrogate for “warm ischemia time”, or in other words, how long were they warm and dead. This should be obtained and sent early in the resuscitation of the patient. If the value is >12, there is nearly no chance of any meaningful recovery (still very unlikely at >10, and even a cutoff of >8). Conversely, if the potassium level is less than the 8-12 range, the patient still has a good chance at a meaningful recovery if resuscitated to ROSC and these are the patients that should receive everything we have to rapidly and efficiently rewarm them (they are also the patients that can have meaningful recoveries despite impressive downtimes of even hours).

Additionally, historical factors surrounding the hypothermia, if known, can provide valuable prognostic information. Immersion vs. Submersion, which you define in your algorithm, is one example that might influence your decision about whether a patient might have benefit from mobilizing ECMO or other aggressive/invasive rewarming.

Screen Shot 2018-05-31 at 11.29.58 AM.png

Kory Gebhardt, MD

Kaiser Permanente Emergency Medicine


How to cite this post

[Peer-Reviewed, Web Publication]   Neil L, Rueter Q (2018, June 4 ). Approach to Hypothermic Resuscitation.  [NUEM Blog. Expert Commentary by Gebhardt K]. Retrieved from http://www.nuemblog.com/blog/hypothermia


Posted on June 4, 2018 and filed under Cardiovascular.

Treatment of pSVT: A Case for Calcium Channel Blockers

Written by:  Amanda Randolph, MD (NUEM PGY-1) Edited by: Jim Kenny, MD, (NUEM PGY-4) Expert commentary by: Meghan Groth, PharmD - Emergency Medicine Clinical Pharmacist, UMass Memorial Medical Center 


The Case

A 37 year-old woman presents to the ED for palpitations. On the monitor, you see her heart rate is 190, but all other vitals are within normal limits. She feels anxious but is otherwise asymptomatic, breathing comfortably on room air. The rest of the physical exam is unremarkable. The patient tells you, “I think it’s my SVT again - I was just here for this last month!”

Her rhythm strip looks something like this:

https://lifeinthefastlane.com/ecg-library/svt/

SVT generally refers to any tachyarrhythmia generated above the His/Purkinje system. For simplicity, the term pSVT in this post will refer to only Atrioventricular Nodal Tachycardia (AVNRT), as it is the most common tachyarrhythmia in patients with normal cardiac structure [1].


SVT: Treatment Guidelines

You double-check the current ACLS protocol (2015) for the treatment of pSVT [2]: 

Figure 1. ACLS 2015 guidelines for treatment of AVNRT

This patient is stable, so you try some vagal maneuvers, including carotid massage and Valsalva. You even try the modified Valsalva maneuver you read about in the REVERT trial (straining followed by leg elevation and supine positioning), which is described to have a 43% success rate.

 

Despite your best efforts, the vagal maneuvers fail, so you ask the nurse to draw up some adenosine. 

 

At this point, the patient yells, “Absolutely no way! I’m not trying Adenosine - it makes me feel like I’m going to die! There has to be something else.” 

 

 

You know Calcium Channel Blockers (CCBs) are recommended as a second line drug if adenosine does not terminate the SVT, or if adenosine is contraindicated. But what does the data say? Is it ever reasonable to jump straight to CCBs?


The Problem with Adenosine

Adenosine administration is widely recognized to produce a variety of minor side effects, as listed below4. While not quantified in any studies to date, these “minor” side effects can be extremely traumatic for patients. This distress can have lasting psychological effects that may delay or even prevent patients from seeking care [5].

  •  Chest pain (7-40%)
  •  Facial flushing (18-44%)
  •  Nausea (13%)
  • Headache (2-18%)
  • Lightheadedness/Dizziness (12%)

The Problem with Calcium Channel Blockers

Current ACC/AHA guidelines give CCBs a class IIa recommendation for use in pSVT [2]. However, most EM practitioners continue to favor Adenosine, in part because of cultural dogma, but also due to concern about inadequate data to regarding the efficacy and safety for calcium channel blocker use. 

One pharmacologic difference between CCBs and Adenosine is the onset of action (100-400 seconds for CCBs compared with 21-34 seconds for Adenosine), which can create a delay to conversion [5]. However, because CCBs are only used in stable patients, this slightly longer onset is unlikely to be clinically significant [6].

More importantly, one of the most feared side effects of Calcium channel blockers is hypotension, as CCBs work by creating negative inotropy and peripheral vasodilation. In one study by Lim et al., the change in blood pressure after administration of adenosine was -2.6/-1.7, compared to -13.0/-8.1 with verapamil [9]. 

Of note, the duration of action is quite long for CCBs (2-5 hours), compared with adenosine (<10 seconds).7 This raises a concern that hypotension and other adverse effects of CCBs may be prolonged. For this reason, CCBs are contraindicated in patients with severe HFrEF.6,7 Additionally, CCBs are relatively contraindicated in patients taking beta blockers, as the combined effect can cause significant bradycardia and even heart block [6].

Theoretically, the use of CCBs via slow infusion instead of IV bolus may reduce the risk of hypotension,8 though there is limited data to support this. One randomized trial by Lim et al. compared the use of adenosine (n = 104) vs slow infusion of verapamil (n = 48) or diltiazem (n = 54), and reported no difference in outcomes between adenosine bolus and slow infusion of verapamil or diltiazem [9].


Calcium Channel Blockers vs. Adenosine - The Data

To date, there have been three meta-analyses comparing the efficacy and safety of CCBs to adenosine in patients with pSVT, including a recently published Cochrane review in October 2017.5,6,10 Note that the data described in these studies only refer to the use of Verapamil. Their findings are depicted below (table design inspired by a phenomenal ALiEM post) [8].

A Few Notes on Hypotension after Verapamil:

  • None of these metaanalyses specifically reported their definition of hypotension, nor did they clarify whether any of these patients had clinical signs of shock.
  • Holdgate and Foo reported two of three hypotensive patients subsequently reverted with adenosine and did not require any other specific treatment for their hypotension (the outcome and interventions for the third case were not reported). 
  • The study by Lim et al. using slow infusion of verapamil reported only one patient with clinically significant hypotension, with a drop in blood pressure from 122/81 mmHg to 74/61 mmHg after 7.5 mg of verapamil infusion. This patient’s SVT was terminated by synchronized cardioversion, after which his blood pressure improved to 103/69 mmHg.

Case Resolution

After the vagal maneuvers, you give 5mg IV Verapamil. The patient remains stable and converts to sinus tachycardia. She tells you she prefers Verapamil to Adenosine and will be “much less afraid” to come in next time. 


Conclusion

Overall, both Adenosine and Verapamil are reasonable choices for termination of SVT. Anecdotally, some patients prefer Verapamil; however, there is limited evidence to support this [6]. Given the current data, physicians should discuss the pros/cons of each drug with the patient and employ shared decision-making when possible. 


Take Home Points

  •  Start with vagal maneuvers, especially the modified Valsalva
  • Adenosine and Verapamil are equally effective for SVT 
    •  Moderate evidence by recent Cochrane review
    •  Class IIa by ACC/AHA
  •  Adenosine has a much higher incidence of minor side effects
    • chest pain, facial flushing, nausea, headache, and lightheadedness/dizziness
  • Verapamil has a slightly higher risk of hypotension
    • Verapamil: -13/-8 mmHg; Adenosine -2.6/-1.7 mmH
    • Rarely clinically significant - cases reportedly resolved with adenosine or synchronized cardioversion
  • Always employ shared decision-making when possible 

Expert Commentary

Thank you for your insightful post on this all-too-common conundrum we face in the ED. I think it’s incredibly important to remember, as you point out, that treatment of pSVT in the ED doesn’t have to be a “one size fits all” approach, and that we have more than just adenosine available as a treatment agent.

Most of the data for CCBs in this indication is with verapamil, but I’ve become comfortable recommending diltiazem in its place due to a lower risk of hypotension (see post for reference).

When attempting to mitigate the potential hypotension associated with calcium channel blockers, the study by Lim and colleagues that you mentioned is worth noting in more detail. Rather than the traditional 0.25 mg/kg diltiazem bolus (with 0.35 mg/kg repeat dose), subjects instead received diltiazem at a rate of 2.5 mg/min for up to 20 minutes (max dose 50 mg). This approach can optimize dose, reduce potential for hypotension, and spare the patient that “impending doom” feeling often experienced with adenosine (see further discussion on this here).

There are also some cases when adenosine should not be routinely administered, such as patients with reactive airway disease at risk of bronchospasm. A more thorough review of this topic is presented here but in such cases calcium channel blockers represent a reasonable alternative.

The strategy of using calcium channel blockers for pSVT can perhaps leave providers wanting in terms of the instant gratification that comes with adenosine administration, but agents like diltiazem or verapamil have demonstrated efficacy while avoiding some of the unpleasantries of adenosine.

For me, it comes down to recognizing that adenosine isn’t the only drug we have available for the treatment of pSVT. Calcium channel blockers like diltiazem may be used, and if we decide to try them, we can use different dosing approaches such as the slow bolus method outlined above to reduce some of the potential side effects.

NO_NAME-30 (1).png

Meghan Groth, PharmD

Emergency Medicine Clinical Pharmacist 

UMass Memorial Medical Center

 


[Peer-Reviewed, Web Publication]   Randolph A,  Kenny J (2018, May 28 ). Treatment of pSVT: A case for calcium channel blockers.  [NUEM Blog. Expert Commentary by Groth, M]. Retrieved from http://www.nuemblog.com/blog/PSVT


Posts you may also enjoy


References

  1. Burns, E., Supraventricular Tachycardia (SVT), in Life in the Fast Lane, M. Cadogan and C. Nickson, Editors. 2012.
  2. Page R, Joglar J, Caldwell M, et al. 2015 ACC/AHA/HRS Guideline for the Management of Adult Patients With Supraventricular Tachycardia: A Report of the American College of Cardiology/American Heart Association Task Force on Clinical Practice Guidelines and the Heart Rhythm Society. J Am Coll Cardiol. 2016;67(13):e27-e115.
  3. Appelboam, A., et al., Postural modification to the standard Valsalva manoeuvre for emergency treatment of supraventricular tachycardias (REVERT): a randomised controlled trial. The Lancet, 2015. 386(10005): p. 1747-1753.
  4. Adenosine. Lexi-Drugs. Lexicomp. Wolters Kluwer Health, Inc. Riverwoods, IL.  Available at:  http://online.lexi.com.  Accessed November 12, 2017.
  5. Delaney, B., J. Loy, and A.-M. Kelly, The relative efficacy of adenosine versus verapamil for the treatment of stable paroxysmal supraventricular tachycardia in adults: a meta-analysis. European Journal of Emergency Medicine, 2011. 18(3): p. 148-152.
  6. Alabed S, Sabouni A, Providencia R, Atallah E, Qintar M, Chico TJA. Adenosine versus intravenous calcium channel antagonists for supraventricular tachycardia. Cochrane Database of Systematic Reviews 2017, Issue 10. Art. No.: CD005154. DOI: 10.1002/14651858.CD005154.pub4.
  7. [Peer Reviewed, Web Publication] S. Brubaker and B. Long (2017 Feb 1). Treatment of Refractory SVT: Pearls and Pitfalls. [EmDocs.net, Expert Commentary by A. Koyfman]. Retrieved from http://www.emdocs.net/treatment-refractory-svt-pearls-pitfalls/
  8. [Web Publication] S. Rappaport and M. Groth (2016 Mar 3).  Calcium channel blockers for stable SVT: A first line agent over adenosine? [AliEm Blog]. Retrieved from https://www.aliem.com/2016/03/calcium-channel-blockers-stable-svt-alternative-to-adenosine/ 
  9. Lim S, Anantharaman V, Teo W, Chan Y. Slow infusion of calcium channel blockers compared with intravenous adenosine in the emergency treatment of supraventricular tachycardia. Resuscitation. 2009;80(5):523-528.
  10. Holdgate, A. and A. Foo, Adenosine versus intravenous calcium channel antagonists for the treatment of supraventricular tachycardia in adults. The Cochrane Library, 2006. 
Posted on May 28, 2018 and filed under Cardiovascular.

A Recipe for Reduction: Five alternative approaches for reducing an anterior shoulder dislocation

Screen Shot 2018-05-01 at 7.11.23 PM.png

Written by:  Abiye Ibiebele, MD (NUEM PGY-1) Edited by: Jacob Stelter, MD, (NUEM PGY-3) Expert commentary by: Andrew Ketterer, MD


“With great power comes great responsibility.”
“That’s one small step for man, one giant leap for mankind.”
“Every emergency medicine physician should know three ways to reduce a shoulder, not including traction-countertraction.”

            Now that last one may be not as well known as the other quotes, but it was a pearl passed along to me during my Sports Medicine rotation by my attending. The traction-countertraction method is often used due to physician familiarity and is considered the standard technique due to a high success rate [1,7] However, due to need for adequate sedation and the amount of force generated during the reduction, below we will examine five alternative methods of reduction for anterior shoulder dislocations.


Stimson Method

Figure 1: Stimson maneuver of shoulder reduction Image credit: http://img.medscapestatic.com/pi/meds/ckb/20/25520.png

  • Have the patient lay prone on an elevated stretcher with the injured extremity hanging off the edge of the stretcher. [1]
  • Apply traction by suspending 5 to 10 lbs of weight from the wrist. [1]
  • Have the patient maintain this position for 20-30 mins. [1]
  • If needed, manual traction can be added with external rotation to aid in reduction. [1]

 

  •  Success rate for the Stimson technique alone is about 28%. [3,4]
  • Success rate improves when combined with scapular manipulation.
  • Reasons for failure include discomfort in prolonged prone position and discontinuing the reduction with prolonged times which can reach over 20 mins. [4]
  •  Moderately painful, ~5.3 out of 10 on pain scale. [3]

Scapular Manipulation Method

Figure 2: Scapular Manipulation Technique.  Adapted from Horn, A., & Ufberg, J. (2013), Management of Common Dislocations. In: Roberts and Hedges' Clinical Procedures in Emergency Medicine (6th ed.). Philadelphia, PA: Elsevier/Saunders.

  • Place the patient in a prone position with the shoulder in 90 degrees of forward flexion and slight external rotation. [1,2]
  • Apply traction to the shoulder as mentioned in the Stimson technique above. [1,2]
  • As patient begins to relax, stabilize the superior aspect of the scapula with one hand, with the thumb on lateral border of scapula. [1,2]
  • With other hand, push the inferior tip of scapula medially towards spine, while rotating superior aspect laterally with the first hand. [1,2]

  •  Some dorsal displacement of the tip of the scapula (lifting it) may be necessary as medial displacement is maximized. [1,2]

 

 

 Variation: This technique can also be done in a seated position, with an assistant assisting applying traction on the affected arm and countertraction on ipsilateral clavicle. This is actually the preferred method by many, however this is a technically more difficult reduction [1].

  • Success rate for the Stimson technique has ranged from ~90-97%. [2,3]
  • Fast reduction, takes less than 5 minutes to perform. [2,3]
  • Noted to be one of the least painful methods of reduction: a recent systematic review describes pain ~1.5 out of 10 during reduction. [3]
  • There have not been any reported complications of this technique. [2,3]

 

External Rotation Method

Figure 3: External Rotation Technique.  Adapted from Horn, A., & Ufberg, J. (2013), Management of Common Dislocations. In: Roberts and Hedges' Clinical Procedures in Emergency Medicine (6th ed.). Philadelphia, PA: Elsevier/Saunders.

  •  Have the patient lie supine on a stretcher and position yourself on the side of the affected arm. [1,5]
  • Fully adduct the affected arm and flex the elbow to 90 degrees. [1,5]
  • Place one hand on the wrist and another hand on the patient’s elbow. [1,5]
  • Using the grasped wrist as a guide, slowly begin to externally rotate the patient’s arm. [1,5]
  • Stop movement any time patient feels pain to allow the muscles to relax before resuming. [1,5]
  • Reduction typically occurs between 70 and 110 degrees of external rotation. [6]

 

  •   If dislocation persists after full external rotation, you can apply steady gentle traction at the elbow, or slowly bring the arm back into internal rotation which can lead to reduction [1,6}.
    •  You can also proceed to the Milch technique from full external rotation (see below). [8]
  •  Success rate ranges from 81-91%. [3,6]
  • Average time to reduction is around 3 mins but it can take up to 10 mins to perform. [3,6]
  • Well tolerated by patients, ~ 3 out of 10 on pain scale. [3]
  • No reported complications of this technique. [3,6]

Milch Technique

  •  Have the patient lie on a stretcher; the patient can be either supine or prone based on his or her comfort. [1,4,6]
  •  Have the patient abduct the affected arm to place their hand behind their head, if they are able, and then straighten the arm at the elbow. [1,6]
    • If the patient cannot do this unassisted, then grab patient’s arm at either the elbow or the wrist and guide arm into full abduction. [1,4,6]
  •   With the arm fully abducted, apply gentle longitudinal traction and gentle external rotation to achieve reduction. [1,4,6]
  •  If reduction does not occur quickly, apply gentle cephalad pressure to the humeral head while continuing to hold traction. [1,4,6]
  •  If external rotation has already been attempted (see external rotation technique above), you can proceed to the Milch technique by abducting the arm in a wide arc from full external rotation, while applying gentle traction throughout. [8]

Figure 4: Milch Technique.  Adapted from Horn, A., & Ufberg, J. (2013), Management of Common Dislocations. In Roberts and Hedges' Clinical Procedures in Emergency Medicine (6th ed.). Philadelphia, PA: Elsevier/Saunders.

  • Success rate ranges from 70-95%. [1,6]
  •  On average, takes about 4-5 mins to perform. [3,4]
  • Moderately painful, ~ 5.3 out of 10 on pain scale. [3,4]
  • No reported complications of this technique. [3,4]

FARES Method (“FAst, REliable, Safe)

  •  Have the patient lie supine on the stretcher and stand on the affected side. [9]
  •  Apply gentle longitudinal traction on the arm and begin to bring the arm into abduction. [9]
  •  While abducting arm, oscillate the arm in an up and down fashion
    • Oscillations should be brief (2-3 full cycles per second) and short (about 5 cm above/below midline). [9]
  •  After 90 degrees of abduction, continue oscillations and add gentle external rotation. [9]
  •  Reduction is usually achieved around 120 degrees of abduction. Afterwards gently internally rotate the arm to bring the forearm to lie across the patient’s chest. [9]
  •  A helpful demonstration video can be viewed here

 

  • Success rate ranges from 88-95%. [9, 10]
  •  Reduction time: ~2-3 mins. [3,9,10]
  •  Well tolerated, pain 1-2 out of 10 on VAS scale. [3,9]
  • No reported complication of this technique. [3.9]

So, the next time an anterior shoulder dislocation walks into the ER, go ahead and give one of these reduction techniques a try. No single reduction method is 100% successful, so it’s good to be facile in a variety of methods. Remember to obtain pre- and post-reduction films and assess neurovascular status before and after reduction [1]. Time to reduce some shoulders!


Expert Commentary

This is a very nice overview of some less brutal approaches to a common and sometimes difficult problem. The classic traction-countertraction techniques (e.g. the Hippocratic method, wherein the physician places a foot in the axilla of the patient’s affected arm and applies distal traction) tend to have higher complication rates, including axillary nerve injury, humeral neck and shaft fractures, and glenohumeral capsular damage. They also tend to be quite painful, usually necessitating procedural sedation, which of course carries its own risks.

In addition to the above, one method I have had great success with is the Cunningham technique: The patient is placed in a sitting position, with the affected arm completely adducted and the elbow flexed to 90 degrees. The physician supports the patient’s forearm with their own forearm, with the hand on the patient’s elbow, and applies very gentle downward traction – the weight supplied by the physician’s forearm is usually adequate. The physician sequentially massages the patient’s trapezius, deltoid, and biceps muscles until the humeral head reduces. This technique won’t usually cause a satisfying “clunk,” so you’ll need to check periodically to see whether the shoulder has been reduced. Resolution of the lateral shoulder step-off might be the only immediately visible sign of successful reduction.

[Video of Cunningham technique]

Often, I will combine this technique with the FARES method by oscillating the patient’s forearm up and down as I externally rotate their shoulder. This usually results in quick and nearly painless reduction and has an exceptionally low complication risk. In order for these techniques to work, the patient must be relaxed – as soon as you hit resistance or cause pain their muscles will tense up, so if this happens you need to pause and wait for them to feel better before continuing. A whiff of opioids can do wonders here, accomplishing both pain relief and anxiolysis.

The reasoning behind the various shoulder reduction techniques is that spasm of the biceps, trapezius, and deltoid muscles is keeping the humeral head out of the glenoid fossa. Fatiguing these muscles with traction or distracting the patient will allow you to mobilize the humeral head and get it back into the glenoid fossa. It’s worth noting that muscle spasm becomes increasingly hard to overcome the longer a patient is dislocated. This means that the FARES method and other distraction techniques are less likely to work if the patient has been dislocated for too long, and more painful fatigue techniques such as Stimson, Milch, or good old traction-countertraction may become necessary. Still, it’s good to have a number of tricks up your sleeve, and if one doesn’t work, you have plenty of others to choose from.

 

Screen Shot 2018-05-14 at 10.44.52 PM.png

 

Andrew Ketterer, MD

Medical Education Fellow, Beth Israel Deaconess Emergency Medicine

 

 


[Peer-Reviewed, Web Publication]   Ibiebele A,   Stelter J (2018, May 21 ). A Recipe for Reduction: Five alternative approaches for reducing an anterior shoulder dislocation.  [NUEM Blog. Expert Commentary by Ketterer, A]. Retrieved from http://www.nuemblog.com/blog/shoulder-reduction



References

1.     Horn, A., & Ufberg, J. (2013), Management of Common Dislocations.In Roberts and Hedges' Clinical Procedures in Emergency Medicine (6th ed.). Philadelphia, PA: Elsevier/Saunders.

2.     Anderson, D., Zvirbulis, R., & Ciullo, J. (1982). Scapular manipulation for reduction of anterior shoulder dislocations. Clinical orthopaedics and related research, 164, 181-183.

3.     Alkaduhimi, H., van der Linde, J. A., Willigenburg, N. W., van Deurzen, D. F. P., & van den Bekerom, M. P. J. (2017). A systematic comparison of the closed shoulder reduction techniques. Archives of orthopaedic and trauma surgery, 137(5), 589-599.

4.     Amar, E., Maman, E., Khashan, M., Kauffman, E., Rath, E., & Chechik, O. (2012). Milch versus Stimson technique for nonsedated reduction of anterior shoulder dislocation: a prospective randomized trial and analysis of factors affecting success. Journal of shoulder and elbow surgery, 21(11), 1443-1449.

5.     Eachempati, K. K., Dua, A., Malhotra, R., Bhan, S., & Bera, J. R. (2004). The external rotation method for reduction of acute anterior dislocations and fracture-dislocations of the shoulder. JBJS, 86(11), 2431-2434.

6.     Ufberg, J. W., Vilke, G. M., Chan, T. C., & Harrigan, R. A. (2004). Anterior shoulder dislocations: beyond traction-countertraction. The Journal of emergency medicine, 27(3), 301-306.

7.     Ghane, M. R., Hoseini, S. H., Javadzadeh, H. R., Mahmoudi, S., & Saburi, A. (2014). Comparison between traction-countertraction and modified scapular manipulation for reduction of shoulder dislocation. Chinese Journal of Traumatology, 17(2), 93-98.

8.     Hendey, G. W. (2016). Managing anterior shoulder dislocation. Annals of emergency medicine, 67(1), 76-80.

9.     Sayegh, F. E., Kenanidis, E. I., Papavasiliou, K. A., Potoupnis, M. E., Kirkos, J. M., & Kapetanos, G. A. (2009). Reduction of acute anterior dislocations: a prospective randomized study comparing a new technique with the Hippocratic and Kocher methods. JBJS, 91(12), 2775-2782.

10.  Maity, A., Roy, D. S., & Mondal, B. C. (2012). A prospective randomised clinical trial comparing FARES method with the Eachempati external rotation method for reduction of acute anterior dislocation of shoulder. Injury, 43(7), 1066-1070.

 

 

 

The PATCH Trial

Screen Shot 2018-05-01 at 7.02.02 PM.png

Written by:  Andrew Berg, MD (NUEM PGY-3) Edited by: Ryan Huebinger, MD, (NUEM Grad 2017) Expert commentary by:  Stephen Trevick, MD


Intro:

Hemorrhagic strokes, while accounting for less than 20% of incident strokes, contribute to half of all stroke-related deaths and long-term disability, totaling up to 47 million life-years lost [1]. Unlike ischemic strokes which have the potential to be intervened upon, non-surgical treatment of hemorrhagic stroke is limited. Hemorrhagic stroke associated with the use of non-reversible antiplatelet agents can be problematic, and the goal should be to try to limit the extent of the hemorrhage. In this day and age of increasing myocardial ischemia, percutaneous coronary intervention, and ischemic stroke leading to increasing antiplatelet usage, the attempted reversal of antiplatelet agents with platelet transfusion seems like a logical step. However, it is unclear if this is efficacious (both in the short and long-term) or if there are potential harms as studies are limited. The PATCH Trial looked at the use of platelet transfusion after acute spontaneous intracerebral hemorrhage in people taking antiplatelet therapy to determine if there was any impact on long-term functional outcomes.


The Study:

Baharoglu, M. Irem, et al. "Platelet transfusion versus standard care after acute stroke due to spontaneous cerebral haemorrhage associated with antiplatelet therapy (PATCH): a randomised, open-label, phase 3 trial." The Lancet 387.10038 (2016): 2605-2613. 


Study Design:

This was a 6-year multicenter (60 hospitals in Europe), parallel-group trial that randomized 190 patients with non-traumatic supratentorial intracerebral hemorrhage while on antiplatelet therapy to standard care or standard care plus platelet transfusion and performed both an intention-to-treat and as-treated analysis of the outcomes.


Population: 

Inclusion criteria:

  1. 18 years or older with non-traumatic supratentorial intracerebral hemorrhage confirmed by brain imaging
  2. Glasgow Coma Scale ≥ 8
  3. Platelet transfusion could potentially be initiated within 6 hours of symptom onset and within 90 minutes of brain imaging
  4.  On antiplatelet therapy with either a COX inhibitor (aspirin or carbasalate), ADP receptor inhibitor (clopidogrel) or an adenosine-reuptake inhibitor (dipyridamole) for at least 7 days prior to the ICH.
  5. Pre-ICH mRS (modified Rankin Score) score of 0 or 1 only, suggesting no prior disability.

Exclusion criteria:

  1. Imaging findings suggestive of epidural or subdural hematoma or those needing surgery within the next 24 hrs of admission
  2.  Imaging suggesting underlying aneurysm or AVM
  3.  Prior adverse reaction to platelets
  4.  Use of a vitamin K antagonist
  5.  Known coagulopathy
  6. Imminent death

 

Patient selection flow chart: 

Screen Shot 2018-05-13 at 10.22.26 AM.png

Intervention protocol: 

The patients were randomized to either standard care or standard care plus platelet transfusion. Both groups had a repeat brain imaging 24 hours after intervention.

Standard care was not defined in the protocol, but was assumed to be given according to contemporary European and national guidelines.

Platelet transfusions were initiated within 6hrs of intracerebral hemorrhage symptom onset and within 90 min of diagnostic brain imaging.  Those patients on clopidogrel received 2 units of platelets, all others received one unit as determined by in-vitro experiments. 

Outcome Measures

The primary endpoint was difference in functional outcome at 3 months after randomization scored with the mRS, as was done for the inclusion criteria, which was scored by a physician or nurse not involved with the medical treatment. Secondary clinical endpoints further stratified these outcomes into survival, poor outcome defined as mRS of 4-6 and poor outcome defined as mRS of 3-6. Another secondary outcome was median absolute intracerebral hemorrhage growth in mL after 24 hours on brain imaging. Safety outcomes and other serious adverse events were also recorded.

Results

For the primary endpoint of functional outcome difference at 3 months, there was an increase in odds toward death or dependence at 3 months in those that received platelet transfusions, both adjusted and unadjusted (adjusted common OR 2.05, CI 1.18-3.56, p=0.0114). Secondary analysis and serious adverse events are listed in the following graphs:


Interpretation

For the primary outcome, there was increased odds of a poorer functional outcome after 3 months among those who received platelet transfusions compared to those who just received standard care. In the secondary analysis (which stratifies these functional outcomes further into smaller categories), showed there was a significant poorer outcome at 3 months within the mRS 4-6 category (higher disability) for those that received platelets. All the other subcategories were insignificant. Median ICH growth did not differ significantly between the two groups at 24 hours. Among the serious adverse events, the only minimally significant difference was an increased odds of an adverse event due to ICH as a whole in the platelet transfusion group, though this may be explained by differences in the baseline characteristics of the different arms of the study.

Strengths

  • While there was some crossover between the two study arms, there was an intention-to-treat analysis as well as an as-treated analysis.
  • Follow-up was strong without attrition.
  • The physician/nurse that performed the mRS after 3 months was blinded to randomization.
  • Their intended inclusion/exclusion criteria were strong.

Weaknesses

  • Looking at the baseline characteristics, there were several patients included in the trial that should have been excluded by criteria (GCS <8, infratentorial ICH location, etc).  
  • Even though the paper states that the baseline patient characteristics were balanced between the two arms, it appeared that as a whole, the patients who were randomized to receive platelets were sicker (lower GCS, more patients with ICH volume >30mL, both the infratentorial ICH patients).
  • The majority of patients were on a COX inhibitor (>90% of patients), with very little representation of the other antiplatelet agents
  • This study could have been strengthened with platelet function testing to evaluate for modified treatment effect.

Internal/external validity

External validity could be questioned as this study presumably included only European patients (although race was not specifically mentioned). This study also likely cannot be generalized to non-COX inhibitors given how few patients were on ADP inhibitors, such as clopidogrel.

Internal validity is questioned with the inclusion of patients who met exclusion criteria and should have been excluded. The authors comment on this being an issue for several emergency-department studies given situational urgency.

Future Directions

Given the high prevalence in use of Plavix or Ticagrelor, there should be a study that includes more ADP inhibitors.


Summary: 

  • This was a European multi-center randomized trial comparing the functional outcomes of patients with spontaneous intracerebral hemorrhage on antiplatelet therapy, when they received either standard care or standard care plus platelet transfusion.
  •  The study included 190 patients from either Netherlands, UK or France.
  •  Their results indicated that platelet transfusion did not benefit patients from a functional outcome after 3 months, and in fact, may be associated with worse outcomes. Although there were several weaknesses in the study’s execution, these results seem to be significant enough to have some validity.
  • There was no difference in reported/observed immediate (<24 hour) outcomes on imaging between the two groups

Expert Commentary:

ICH is a devastating disease, and often one which we often must watch powerlessly, despite the acuity of presentation.  One of the first goals in ICH management is to prevent further bleeding.  Platelets for aspirin reversal seemed promising, since it has been documented that patients on aspirin have more hematoma expansion and worse outcomes, as well as clear anecdotal evidence from surgeons that platelet infusion in aspirin users makes an overt difference intra-op.  So why should PATCH have been negative?

Given the complexity of factors leading to hematoma expansion and subsequent hospitalization, it is hard to drive outcomes with any one intervention.  However, even the rate of hematoma expansion was unchanged.  Even though the irreversible binding of Aspirin usually takes many days to wash out, serum half-life is about 15-20 minutes (with active metabolites lingering a few hours).  One thing to remember is that transfused platelets themselves can have time-limited efficacy due to immune-related consumption and inactivation.  While even a temporarily effective transfusion can help stop brief bleeding such as in the OR or during acute stabilization, it is unlikely to prevent the stuttering hematomal expansion of ICH.

Platelets, unlike other blood products, must be stored at room temperature.  Therefore, platelet transfusions are associated with high risks of transfusion reactions.  It is theorized that some platelets may become activated prior to transfusion and can therefore also be associated with a risk of pathological clotting.  These factors could blunt any potential benefit of treatment.

Intracerebral hemorrhage is a rarer and more heterogenous illness than stroke or MI.  This trial is as high of quality as we are likely to obtain on the topic.  When applying the results of the PATCH trial, it is important to remember that no patients with platelet counts below 100k were enrolled, so transfusions to meet that goal may still be performed.  Also, it is still acceptable to transfuse for procedures or surgery.  The study did not explore any sort of functional assays, namely Platelet Function Assay (PFA) or Platelet Aspirin Assay (PAA). 

 

NO_NAME-30 (1).png

Stephen Trevick, MD

Neurocritical Care Fellow, NUEM

 


How to cite this post

[Peer-Reviewed, Web Publication]  Berg A,   Huebinger A  (2018, May 14 ). The PATCH Trial.  [NUEM Blog. Expert Commentary by Trevick, S]. Retrieved from http://www.nuemblog.com/blog/PATCH


Posts you may also enjoy


Resources

1. Feigin VL, Krishnamurthi RV, Parmar P, et al. Update on the global burden of ischemic and hemorrhagic stroke in 1990–2013:the GBD 2013 Study. Neuroepidemiology 2015; 45: 161–76.

Posted on May 14, 2018 and filed under Neurology.

A Visual Guide to Upper Extremity Joint Aspirations

Screen Shot 2018-05-01 at 6.43.26 PM.png

Written by:  Will Ford, MD (NUEM PGY-3), Amy Ford, MD (Loyola Orthopedic Surgery PGY-3)  Edited by: Keith Hemmert, MD, (NUEM PGY-4) Expert commentary by:  Lucas Rosiere, MD


What follows is an overview of joint aspiration techniques in the upper extremity.  We will be covering the shoulder, elbow, and wrist.


THE SHOULDER

Coracoid

Identify your landmarks.  In this setting, the skin marker is your friend.  See the following pictures.

1. Find the coracoid.  Mark it with a circle.

2. Find the notch where the acromion and the clavicle meet.  Mark it with a point.

3. Find the anterolateral and posterolateral corners of the acromion.  Mark each with a point.  These two points should make an equilateral triangle with your point from #2.

 

From L to R: Anterolateral corner of acromion, meeting between the acromion, posterolateral corner of the acromion

 

4.     Roll your marker off the lateral edge of the acromion.  Mark this line.

5.     Draw out the borders of the clavicle and the scapular spine.

 

Two common sites of entry are the posterior and anterior approaches.  You will find that some patients have better landmarks posteriorly or anteriorly – this can vary depending on patient habitus and positioning. 

Posterior approach

The posterior approach traditionally begins approximately 2cm medial and inferior from the posterolateral corner of the acromion.  You should feel a soft spot here.  In a patient without a septic joint, you can move the humeral head anteriorly and posteriorly and feel the joint from this spot.  When aspirating from the posterior approach, go in through the soft spot and point your needle toward the coracoid circle you marked previously.

The anterior approach uses your coracoid landmark to protect yourself from injuring important neurovascular structures.  Never go medial to the coracoid.  Your insertion spot will be just lateral to the coracoid.  Aim directly posterior and slightly superior.

Anterior approach 

When dealing with shoulder injections with distorted anatomy (for example, in the shoulder dislocation), it can still be helpful to draw out the anatomy and imagine where it will be easiest to enter the joint.  For instance, in an anterior shoulder dislocation, you may be able to palpate the humeral head anteriorly and easily enter the joint space around the humeral head.


The Elbow

Identify your landmarks.  Again, the skin marker can be helpful to visualize your entry point.

  1.  With the elbow at 45-90° flexion, mark the lateral epicondyle of the humerus with a point.

2. Mark the radial head with a point.  You can identify this by pronating and supinating the forearm and feeling the rotation of the radial head.

3. Mark the tip of the olecranon with a point.  The three points should make something close to an equilateral triangle.

 

Your entry point will be in the center of the triangle.  Again, you should feel a soft spot here.  When directing your needle, orient it perpendicular to the skin and go straight in.


The Wrist

Identify your landmarks: 

  1. Find Lister’s tubercle (tubercle on the dorsal distal tip of the radius).  Your entry point will be approximately 1cm distal to this tubercle. 
  2. Find the extensor tendons of the thumb and index finger (extensor pollicis longus and extensor digitorum communis/extensor indicis proprius).  Your entry point will be between these tendons.

You should feel a soft spot at the wrist joint between these landmarks.  It may be helpful to flex the wrist slightly (15-30°) and to point your needle proximally (30-45°) to respect the slope of the distal radius.  Another helpful trick is to have an assistant grab the patient’s forearm with one hand and the patient’s index and middle fingers with the other hand, and distract across the joint to open up the space.  Alternatively, you could hang the arm up in finger-traps for the same effect.

"Arthrocentesis & Injections: Wrist (Radiocarpal)." RheumaKnowledgy Arthrocentesis Injections Wrist Radiocarpal Comments. N.p., 15 Oct. 2014. Web. 14 May 2017. 


General tips for joint aspiration:

  • To produce greater suction power needed to aspirate viscous fluid, use a larger bore needle and a smaller syringe. 
  • If there is any question about length of needle needed to reach the joint, use a spinal needle to avoid multiple attempts.  However, in smaller joints, a shorter needle is preferable because it will improve your proprioceptive senses.
  •  If you enter and hit bone, don’t panic, just try to visualize the anatomy and redirect your needle gently.
  •  If anesthetizing the skin/subcutaneous tissue prior to aspiration, be careful to stay subcutaneous, as injecting lidocaine into the joint space would jeopardize the accuracy of your cell count results.
  •    Lab tests to order on synovial fluid:
    •  Culture + gram stain (most important)
    •  Cell count
    • Crystals

General indications for joint aspiration:

  • The most absolute indication for aspiration is concern for septic arthritis, as evidenced by:
    • Extreme apprehension from the patient to move the joint, such that only passive motion is possible
    • Very limited motion due to pain
    • Painful throughout entire arc of motion (i.e. no painless arcs of motion)
    • Presence of effusion
  • Aspiration may be done to obtain crystals for a diagnosis of gout.
  • Therapeutic aspiration of a hemarthrosis should not be done routinely, and should only be used as a last resort if noninvasive measures (immobilization, compression, ice, analgesics) have failed.

General contraindications for joint aspiration:

  • Aspiration through cellulitis should generally be avoided due to the risk of seeding an uninfected joint.
  •  Aspiration of prosthetic joints should not be performed by the Emergency Medicine provider for the same reason as above, and the decision on whether or not to do so should be deferred to Orthopaedic Surgery.
  • Anticoagulation is not a contraindication for aspiration.

Expert Commentary

Thanks Dr. Ford and Dr. Hemmert for this procedure guide.  

A thorough understanding of these techniques is essential to the general emergency physician.  Outside of an academic center, you will be the one aspirating joints in the ED (with the exception of the hip).  Rare is the day you'll have your orthopedist in the department for help with this diagnostic part of the work-up.  So it is imperative we can safely and efficiently get this done.

This review focuses a lot on anatomy and skin markers.  Rightly so.  Much like a lumbar puncture, the more time you spend on accurate positioning and palpation, the fewer times you'll poke, the less pain you'll cause, and the more likely you are to produce an atraumatic aspiration.  

I urge you to feel these bony landmarks on yourself.  If you don't do this every day, this can be difficult.  Even on the most slender of people (and few patients fit that description), feeling a coracoid process or radial head may be difficult.  

Whether I can feel the landmarks perfectly or not, I love to spend 60 seconds and use ultrasound.  It can find bones deep in the soft tissues of the obese, it can find surprising locations of joint spaces and can also give you a better idea of the trajectory your needle ought to pass.  Great for patient satisfaction.  Very simple.  Just use the linear probe and place it across the expected joint line so you can see bone on left, bone on right, space in the middle.  Then mark the skin.  No need to use it during the aspiration itself, just while marking the skin.

Regarding the specific techniques listed here, the descriptions are fantastic.  I can only add that, for the wrist, you will also like to avoid puncturing the extensor carpi radialis brevis tendon.  If you have the patient actively extend the wrist a little (may be painful), you should feel the tendon between the tendons of the extensor digitorum and extensor pollicus longus.  I'd go just ulnar to that.

Screen Shot 2018-05-02 at 2.59.17 PM.png

 

Lucas Rosiere, MD

NUEM Graduate 2012, Physician Central DuPage Hospital


How to cite this post

[Peer-Reviewed, Web Publication]  Ford W,   Hemmert K  (2018, May 7 ). A visual guide to upper extremity joint aspirations.  [NUEM Blog. Expert Commentary by Rosiere, L ]. Retrieved from http://www.nuemblog.com/blog/joint-aspiration


Posts you may also enjoy


The Migraine Cocktail: Emergency Department Management of Headaches

Screen Shot 2018-03-05 at 8.47.44 AM (1).png

 

Written by: Vidya Eswaran, MD (NUEM PGY-2) Edited by: Danielle Miller, MD, (NUEM PGY-3) Expert commentary by:  Seth Trueger, MD, MPH


Expert Commentary

Thanks for this great overview over ED headache management. Our approach to headaches has matured in recent years, largely because of a bunch of great studies by Ben Friedman’s group at Montefiore (COI: his brother and I were residency classmates); see Headache guidelines (he’s the first et al in the Orr paper cited above, REF); his Annals Expert Clinical Management paper [REF]; and his FOAM post at ALiEM.

My general approach to headaches:

First: Is there a dangerous cause?

Is the headache similar to their prior headaches in character, location, magnitude, timing, associated symptoms? Are there concerning features (exertional, vomiting, personal or family history of aneurysms)? Was it maximal at onset (or sudden/severe)? I find it helpful to ask: “what were you doing when it started?”; “how bad was it when it started?”; “when was it the worst?”; and only after listening for a while, I backdoor into whether it is typical for them (e.g. “it’s usually on that side and you’re nauseated when you get a headache like this…?”). I’ve gotten myself in trouble by asking up front if it’s the same as usual or “worst headache of your life” – even if they don’t mean to, patients sometimes seem like they are trying to validate why they came to the ED (to us or to themselves). Of course, none of these questions are black and white and there’s a lot of room for clinical judgment; one minor deviation from typical headache does not mandate imaging. Patients come to see us for our expertise and often find it reassuring that we’ve listened and examined them and aren’t concerned.

Second: Symptom management

Turn off the lights

Even migraines without frank photophobia often feel better in a darker room. I usually turn off the lights as soon as I walk in. No reason to wait.

Metoclopramide

Any of the dopaminergic antiemetics are effective; I generally use whichever is typically used in my ED (and doesn’t have to come from pharmacy). IM works fine if the patient doesn’t have an IV – most headache patients don’t need labs or IV fluids so no reason to start one routinely and a lot of even severe headaches are suitable for fast track. But they just don’t seem to work PO.

APAP/NSAIDs

I make sure the patient is up to their appropriate daily dosing of acetaminophen or ibuprofen; with appropriate consideration of contraindications, they might make a difference so why not? PO ibuprofen is likely as effective as ketorolac, and getting a shot doesn’t seem to have a placebo effect (if nothing else, this study is worth reading for the amazing design [REF]; summarized in an accompanying editorial on placebos [REF]).

Steroids

Steroids don’t fix the headache today but they decrease recurrence in some patients. I don’t give them to everyone, but for patients who get headaches in groups, have been having headaches for a while, or are just miserable enough, I give 10mg of dexamethasone.

No diphenhydramine

Diphenhydramine doesn’t work for headaches [208 patient RCT, REF].

Diphenhydramine doesn’t prevent metoclopramide-associated akathisia [REF; REF] (which in my experience, is much less common than the literature describes). Midazolam has some effect for prophylaxis [REF], but the rate of akathisia is low enough that I don’t think it’s worth the risks (or extending LOS due to zonking out the patient). [n.b. the same for avoiding prophylaxis for ketamine sedations; REF] If the patient gets akathisia [or an emergence reaction], then I give midaz.

I’ve heard people suggest diphenhydramine works by knocking out the patient and as any migraine sufferer knows, the best treatment is probably sleep, but the evidence suggests that diphenhydramine just doesn’t add much.

That being said, I pick my battles and I don’t knock it our of nurses hands or reprimand the residents every time or even fight with patients if they really think it helps. But never push IV Benadryl – it gets you high [REF].

No fluid

Unless the patient has been vomiting a lot or has another reason to be volume down, there is little reason to give IV fluids [REF], and in my experience, it locks the patient’s LOS into at least however long the bag takes to drip in, and of course patients who need fluids the least are most likely to have the most positional IVs….

Triptans

I’ll be honest, I don’t give triptans, except in the rare cases where patients know they work for them. Usually the patient’s already taken their home dose, and it seems like most patients have missed the window for effectiveness by the time they’re seeing me. I admit that I’m behind the science and this probably has more to do with practice patterns during my training. I’m open to being convinced.

Sleep

If the patient falls asleep and my ED has the bandwidth, I rarely wake them up. Seems worth giving up the room for a few hours for what’s essentially curative therapy for a miserable condition.

Occipital Nerve Block

If the headache is occipital and radiates forward at all, it could be occipital neuralgia, which often responds quite well to occipital nerve blocks. The diagnosis is not very scientific, but the potential treatment is very simple and, if nothing else, just takes a few minutes of my time and a little pain for the patient, which might provide them hours of relief. I recommen a bupivicaine occipital nerve block to any patient who I think could have occipital neuralgia, review the anatomy, make sure I'm not in the artery, and put in a nice dose of bupivicaine.

No opioids

Opioids don’t work for headache. I never* give opioids for headache. Don’t give opioids for headache.

That being said, the evidence base is surprisingly weak (see REF). My personal experience resonates here; I’ve had a handful of classic migraines, and some were when I was studying abroad in Australia where codeine is OTC. It made me sleepy-ish but didn’t help the headache at all, for whatever that’s worth.

*Rarely, if I see a patient with a pain contract that includes opioids for headaches from a reputable source, I don’t die on that hill.

2nd round: magnesium + (metoclopramide or haloperidol)

If the patient need a second round (or if their headache was terrible to begin with) I throw some IV mag at them. The evidence is weak at best, but it might help and is pretty safe, so why not.

I usually re-dose metoclopramide at this point, but if their current or prior headaches are generally refractory, I often switch to haloperidol (2.5mg IV or IM).

Dispo

Key points in communicating with the patient and evaluating them for discharge:

I don’t have a silver bullet to fix their headache. My goal is to make sure we’re not worried that there is something dangerous going on (“good news! we’re not”) – it’s safe to go home and we have a good, safe plan for follow up (we’re not just kicking them out). While I can’t make the headache go away completely, “my other goal is to get you to the point where you can be miserable here or miserable at home” and we can safely discharge with return instructions and follow-up; I consider Neuro or headache specialist referral if it seems appropriate.

I really think that articulating a lot of these steps is helpful. Much of what we do implicitly is not clear to the patient – our across-the-room gestalt, our assessment and thought processes. Patients want to be listened to, they want to know what to expect, they want to know what to do, and they want a doctor who cares.

Screen Shot 2018-04-27 at 6.00.21 PM.png

 

Seth Trueger, MD, MPH

Assistant Professor, Northwestern Emergency Medicine

 


How to cite this post

[Peer-Reviewed, Web Publication]  Eswaran V,   Miller D  (2018, April 30 ). The Migraine Cocktail: Emergency Department Management of Headaches.  [NUEM Blog. Expert Commentary by Trueger, S ]. Retrieved from http://www.nuemblog.com/blog/headache


Posts you may also enjoy


Posted on April 30, 2018 and filed under Neurology.